2018 Petersons ACT Prep Guide 3

You might also like

Download as pdf or txt
Download as pdf or txt
You are on page 1of 88

Practice Test 3

ENGLISH TEST
45 Minutes—75 Questions

DIRECTIONS: In this section, you will see passages with words and phrases that are underlined and numbered. In the
right-hand column you will see a question corresponding to each number that provides alternatives to the underlined part.
In most of the items, you are to choose the best alternative. If no change is needed, choose NO CHANGE.

In other items, there will be either a question about an underlined portion of the passage, or a question about a section of
the passage, or about the passage as a whole. This question type will correspond to a number or numbers in a box.

For each question, choose the best alternative or answer and fill in the corresponding circle on the answer sheet.

Practice Test 3 — english


PASSAGE I

The Names of Flowers

[1]

I look and staring at the first green shoots sprouting up 1. A. NO CHANGE


1 B. look and stare
through the dead leaves. The sight sets me trembling
C. looking and staring
D. looking and stare

with anticipation, and I kneel toward the earth to make sure 2. F. NO CHANGE
2 G. to anticipate
H. and anticipating
J. OMIT the underlined portion.

I’d seen correctly. Yes, the daffodils have already begun 3. A. NO CHANGE
3 4 B. I’ve seen
pushing toward the sun. Like in a fever, I forget my work and
C. I can see
wander about the garden inspecting the mulched beds. D. I will see

4. F. NO CHANGE
G. have beginning
H. will soon began
J. soon beginning to

Winter is almost over, I can taste the coming delirium of 5. A. NO CHANGE


5 B. over and I
flowers.
C. over, and I
D. over, yet I
[2]

Gardening is something new to me, a delight this city boy

never imagined. As a child growing up in an apartment in New

York, it seems enough to know that there were flowers and trees. 6. F. NO CHANGE
6 G. does seem
H. did seem
J. seemed

CONTINUE
  819
Peterson's ACT® Prep Guide 2018
Practice Test 3

Culture marked even more than the seasons were by nature. 7. A. NO CHANGE
7 B. Even culture was marked more than the seasons
Autumn was when we played football in the street, or when I
than nature.
was older, tried to pick the peak foliage weekend to go camping. C. Even the seasons were marked more by culture
than by nature.
We were familiar with the highlights that marked each season,
D. Seasons were marked more even by culture than
by nature.

though not the subtle particulars, the gradual way one trans- 8. If the writer were to delete the underlined portion,
8 the paragraph would primarily lose:
formed into another. Such intimacy was reserved for buildings, F. proof that the author did not understand the
8 seasons.
crowds, and subway trains. 9 G. an example of a particular with which the author
was not familiar.
H. a poetic turn of phrase that makes the essay
more appealing to read.
J. a key transition between two different yet related
ideas.

9. The purpose of Paragraph 2 as it relates to the


remainder of the essay is primarily to:
A. provide background that helps to explain the
significance of the garden to the writer.
B. provide background that explains why the writer
feels lucky to have escaped the city.
C. portray the writer’s childhood as vividly as the
writer’s garden.
[3]
D. establish the writer’s sensitive personality and
ability to appreciate flowers.
Now the twilight of summer is marked for me by budding

chrysanthemums spilling over the retaining wall at the side of

the house. In too seasons, they’ve grown huge and sprawling. 10. F. NO CHANGE
10
G. too season’s
H. to seasons
J. two seasons
At the height of autumn, they are as bright yellow and red and 11. A. NO CHANGE
11 B. bright yellow red and purple
purple as any foliage, and the season ends when they turn brown
C. bright, yellow, and red, and purple
11
and the last rose buds on the climbers leaning up the porch fail D. bright yellow, and red, and purple

to open.

[4]

Daffodils along the front walk mark the beginning of a long 12. F. NO CHANGE
12 G. have marked
Missouri spring that unfolds with crocuses and tulips, irises and
H. marks
peonies under my study window, and forsythia and spirea around J. marked

the edges of the lawn. Summer means daisies swaying on the

hill, and later, black-eyed susans jostling along the fence.

  820
www.petersons.com
Practice Test 3

[5]

At thirty-five, I’m beginning to learn the names of flowers,

and more than just the names. Names are our entry into the

world, and I feel a fresh side of myself come alive as I become

familiar with the words standing for all those vivid scents and

colors springing from the ground. It’s nice to know that one can

keep on growing. finding enough space inside for gardens and 13. A. NO CHANGE
13 B. growing finding
for subway trains.
C. growing, finding
D. growing Finding

Questions 14 and 15 ask about the passage as a whole.


14. Suppose the writer’s primary purpose had been
to provide a basic explanation of the timeline of a
chrysanthemum’s life. Would this essay accomplish that
purpose?
F. Yes, because it describes how chrysanthemums
look at different points in their life cycle.
G. Yes, because it explains when to plant chrysan-
themums, when they bloom, and when they die.
H. No, because the author is more concerned with
explaining personal experiences than gardening
tips.
J. Yes, because it uses the changing times of season
as a way to describe when chrysanthemums bud,
when they bloom, and when they die.

15. The writer wishes to insert the following detail into the
essay:
I could tell you there were pines, but not distinguish
them from cedar.

The sentence would most logically be inserted into


Paragraph:

A. 1.
B. 2.
C. 3.
D. 4.

CONTINUE
  821
Peterson's ACT® Prep Guide 2018
Practice Test 3

PASSAGE II

Pasta and Tomatoes

[1]

As trade goes, so also goes the world. In these days of global 16. F. NO CHANGE
16 G. so therefore,
markets where people and goods crisscross the world, the idea
H. also
J. so

that a development in Asia can have a major affect on America 17. A. NO CHANGE
17 B. effect on
C. effect over
D. affect over

is taken for granted. Less widely understood was the fact that 18. F. NO CHANGE
18 G. is
exchange has always been a major force in world affairs.
H. had been
J. could be
[2]

19 Agricultural techniques developed in the Near East spread 19. Which of the following sentences, if inserted at this
point, would provide the most effective transition to
deeper into Asia, as well as Europe and Africa, evolving form as
the second paragraph?
A. Throughout history, agriculture has always
played a major role in the development of
civilizations.
B. Throughout history, the movement of people
and goods has been a major factor in changing
human societies.
C. Many examples of this can be found by studying
the history of trade over the past 100 years.
D. However, this phenomenon was never as
important as it is in the modern world.

they went. Goods, religion, knowledge, all of these things have 20. F. NO CHANGE
20 G. Goods, religion, knowledge: all of these things
moved about through the ages, adopted here by one people,
20 have moved
forced on another people there, and every wealth of interchange H. Movement has occurred in goods, religion, and
knowledge
also witnessed a wealth of transformation.
J. Goods, religion, and knowledge have all moved

[3]

That is why Herodotus could marvel at the different practices

he found upon his tour, of the ancient Mediterranean, and why 21. A. NO CHANGE
21 B. tour—of
C. tour; of
D. tour of

  822
www.petersons.com
Practice Test 3

too his impressions had been passed on to generations born 22. F. NO CHANGE
22 G. have been
H. are
J. will be

two millennia later. In fact, the ancient Greeks, whom were well 23. A. NO CHANGE
23 B. that
aware of the influences of Egypt on their civilization, did not pass
C. who
their wealth of knowledge directly on to the rest of Europe, which D. which

claims Greece as its root. Greek thought was kept alive by Arab

scholars at the height of Islam’s power and Greek texts had to 24. F. NO CHANGE
24 G. power, and Greek
be translated from Arabic into Latin before the likes of Thomas
H. power, but Greek
Aquinas could open the intellectual door to the European J. power, therefore Greek

Renaissance.

[4]

But perhaps the clearest examples of this exchange and

transformation lay in a realm less heady and much closer to the

stomach. How would modern Italy be without pasta and 25. A. NO CHANGE
25 B. What would modern Italy be like
C. Where would modern Italy be
D. Would modern Italy be

tomatoes? Imagine Switzerland without chocolate or Ireland and 26. F. NO CHANGE


26 G. Can you imagine
H. It’s like imagining
J. Isn’t it strange to imagine

Eastern Europe without potatoes. 27 27. Which of the alternatives provides the most logical
conclusion for Paragraph 4?
A. Yet none of these familiar staples was known in
the Europe of the Middle Ages.
B. These staples have become part of the very
identity of these nations.
C. They would not be the same countries that they
are today.
D. Cuisine is an important element of every culture.

CONTINUE
  823
Peterson's ACT® Prep Guide 2018
Practice Test 3

[5]

Marco Polo brought pasta back to Europe from China.

Tomatoes, potatoes, and cacao, they were all brought back from 28. F. NO CHANGE
28 G. cacao,
H. cacao
J. cacao, which

the Americas and transformed into something else. Therefore, 29. A. NO CHANGE
29 B. Because
C. Since
D. For

while neither tomatoes nor pasta is originally Italian, one cannot 30. Which of the alternatives would conclude this
30 sentence so that it supports the writer’s point about
think of pasta and tomato sauce in all their glorious forms without
30 culture?
thinking of Italy. F. NO CHANGE
30
G. they have been adapted by Italians.
H. they are now important in Italian cooking.
J. one cannot forget that they were once unknown
in Italy.

PASSAGE III

The Life of Aspirin

[1]

There are so many innovations and inventions in our lives

that we take for granted there are even things that are a part of 31. A. NO CHANGE
31 B. granted; there
our daily lives that we have little or no idea about, including
C. granted there;
where they came from or how they developed, although we’re D. . Granted there

happy that they exist and that we have access to them. 32 32. The author of this passage would like to add a
sentence to the end of Paragraph 1, which would help
transition to Paragraph 2. Which of the following
sentences would best serve that purpose?
F. Take, for instance, aspirin.
G. Perhaps that means we’re ignorant.
H. The truth is, some things are better unknown.
J. The Internet is a good place to research new
things.

[2]

Little serious thought is given; to these humble, unassuming 33. A. NO CHANGE


33 B. given to
little pills by the average person, although billions are bought
C. given, to
and sold each year and you’d be hard pressed to find someone D. given. To

  824
www.petersons.com
Practice Test 3

who doesn’t have some in his or her house. These simple-looking

little discs have been responsible for easing countless and innu- 34. F. NO CHANGE
34 G. countless
merable headaches, aches, and pains around the world, yet most
34 H. innumerable or countless
J. many and several

of us don’t know much more about it beyond the fact that there’s 35. A. NO CHANGE
35 B. him
C. us
D. them

likely a bottle full of them somewhere in our medicine cabinets.

Let’s give aspirin the respect its due by taking a closer—look! 36. F. NO CHANGE
36 G. closer—look.
H. closer; look.
J. closer look.
[3]

Aspirin has taken on different forms over its long and illus-

trious history, dating back over 2,400 years to salicin extracts

from the bark of the Willow Tree, which are metabolized by the 37. A. NO CHANGE
37 B. Willow tree
C. willow tree
D. willow Tree

human body into salicylic acid an active ingredient of aspirin. 38. F. NO CHANGE
38 G. (an active ingredient of aspirin)
Evidence of this has been recorded in ancient Egyptian, Assyrian,
H. an active ingredient of aspirin
and Sumerian texts. J. “an active ingredient of aspirin”

[4]

Through the centuries that followed, chemists sought to

improve the effectiveness of salicylic acid, reduce its side effects,

and make it easier to mass produce as a medicine for widespread

public use. In the late nineteenth century, acetylsalicylic acid was

studied as a possible replacement for salicylic acid and, in 1899,

the modern aspirin was born and named by the pharmaceutical

company Bayer.

CONTINUE
  825
Peterson's ACT® Prep Guide 2018
Practice Test 3

[5]

Today, aspirin is perhaps the most widely used medicine on

the planet, and exceeds 50 billion pills estimates the consumption, 39. A. NO CHANGE
39 B. 50 billion estimates the globe—annual con-
globally and annually. This nonsteroidal anti-inflammatory drug
39 sumption of excess pills
(NSAID) is used to treat a wide array of symptoms, inflammatory C. the globe annually consumes 50 billion esti-
mated pill excess
conditions, and illnesses. These include fevers, blood clots, head-
D. estimates of global annual consumption exceed
aches, rheumatoid arthritis, and everyday aches and pains. In 50 billion pills

specific low doses, aspirin has also been suggested as a preven-

tative medicine to reduce the risk of strokes and certain types

of cancer.

[6]

Despite its impressive array of beneficial effects, over the

years weve learned, largely through trail and error, that there are 40. F. NO CHANGE
40 41 G. weave learned
some common unwanted side effects connected with aspirin
H. we’ve learned
consumption. These include stomach pain and bleeding, wors- J. we’ve leaned

ening of asthma symptoms, and ulcers. Aspirin can also be 41. A. NO CHANGE
B. trial and error
C. trail and era
D. tryer and error

hazardous for women who are pregnant or those who drink

excessive amounts of alcohol. 42 42. In Paragraph 6, should the final sentence be omitted?
F. Yes, because it repeats information stated earlier
in the paragraph.
G. Yes, because the paragraph has already provided
sufficient evidence that aspirin can have
dangerous side effects.
H. No, because it is the paragraph’s most convincing
evidence that aspirin can be dangerous.
J. No, because it adds relevant information.

[7]

Here’s an interesting bit of aspirin trivia: Humans aren’t the

only creatures who benefit from the medicinal effects of aspirin.

Veterinarians sometimes administer aspirin to a wide array of

  826
www.petersons.com
Practice Test 3

animals, including dogs and horses, as an anticoagulant and pain

reliever. 43 44
43. The author of this passage has just discovered that
aspirin, when taken at regular low doses, can help
reduce the risk of heart attacks. To which of the
following paragraphs should the author add this
information?
A. Paragraph 3
B. Paragraph 4
C. Paragraph 5
D. Paragraph 6

44. The author of this passage has decided to add a


concluding sentence to this passage. Which of the
following would be the most effective choice?
F. The next time you reach for an aspirin, remember
that you’re about to have a tiny bit of medicinal
history and innovation!
G. There are lots of other things you can take if
you’re feeling ill; aspirin is not your only option!
H. Aspirin is among the most easy-to-purchase
medicines, and you’ll find it in most small grocery
stores, supermarkets, and pharmacies!
J. Can you think of any other significant medical
innovations that have changed our daily lives?

Question 45 asks about the passage as a whole.


45. The author of this essay was recently given an
assignment to research the history of medicines in the
ancient world. Does this essay fulfill this requirement?
A. Yes, because this essay discusses a medicine that
has existed for a long time.
B. Yes, because aspirin was used by the ancient
Egyptians, Assyrians, and Sumerians.
C. No, because this essay isn’t comprehensive enough
PASSAGE IV to fulfill the scope of the assignment.
D. No, because aspirin wasn’t invented until the
A Professional Lesson twentieth century.
[1]

When I first began working as a journalist in the 1970s, there 46. F. NO CHANGE
46
are few women in the field who are taken seriously. I had no G. there are few women in the field who is taken
46 seriously.
H. there were few women in the field who are taken
seriously.
J. there were few women in the field who were
taken seriously.

CONTINUE
  827
Peterson's ACT® Prep Guide 2018
Practice Test 3

illusions, I would have to prove, myself again and again. 47. A. NO CHANGE
47
B. prove
C. prove;
D. prove—
[2]

Editors were surprised when my work turned out to be first

rate. It took me a long time to understand that the reason had

to do with the way I carried myself, not to mention the fact that 48. F. NO CHANGE
48 G. not to mention the fact that I made no attempt
I was making no attempt with which to conceal my anxieties.
48 to conceal my anxieties
I had not yet learned to put on a professional face. H. with the fact that I made no attempt to conceal
my anxieties
J. I made no attempt to conceal my anxieties
[3]

Editors who would try to hand me the softer stories, or the 49. A. NO CHANGE
49 B. Editors would try to hand me the softer stories,
C. The softer stories were the ones editors would try
to hand me,
D. Editors who tried to hand me the softer stories,

stories with a “woman angle.” “Maybe you could do a piece on 50. F. NO CHANGE
50 G. could do the charity work of
the charity work of the First Lady.” I was undaunted by these
50 H. could be doing it on the charity work of
incidents, and by the inevitable sexist jokes and innuendoes. J. could do a piece on charity work for

After all, I had chosen a career that meant not breaking ground 51. A. NO CHANGE
51 B. breaking ground in
in a traditional male bastion. Determination and a thick skin were
51 C. it broke ground in
mandatory. D. broken ground in

[4]

There was, however; an aspect of professional life for whom 52. F. NO CHANGE
52 G. for which
H. for what
J. for why

many of us were not prepared. I remember that when I will 53. A. NO CHANGE
53 B. I was working
be working on a difficult assignment and I checked in with the
53 C. I will have worked
editor, I would tell him about my worries, describe the obstacles D. I will work

I had yet to surmount, or sometimes even complain about the

minor frustrations that had made for a bad day. I thought of it

as communication, being honest in my work. I never doubted

that I would get past these problems. They merely represented

  828
www.petersons.com
Practice Test 3

the moment-to-moment process of doing my job. After all, life

was like that too—full of difficulties that I discussed with friends

as a way of getting through them. 54 54. Would it add to the effectiveness of the essay if the
writer inserted a paragraph at this point describing
the way in which she discussed problems with a
close friend?
F. No, because the writer’s description of how she
interacted with her editor is already clear.
[5] G. No, because such a description belongs just
before this paragraph.
I noticed, therefore, that when my male colleagues spoke H. Yes, because the writer needs to more fully
to editors, no matter what doubts they had, they always said that illustrate how she interacts with other people.
J. Yes, because the writer needs to show why she
everything was under control. It seemed like lying. What were wanted the “softer stories” she was assigned.

these men afraid of. I didn’t realize that by behaving the way in 55. A. NO CHANGE
55 56 B. afraid of
which I’d been acting, my honesty was creating a negative
56 C. afraid of?
impression with my editors. Editors assumed that if I was speaking D. afraid of—

about my difficulties, then I must really be in trouble. Like all 56. F. NO CHANGE
G. my behavior and
H. my behaving in that way and
managers, they wanted to know that everything was under J. OMIT the underlined portion.

control. By sharing the struggles that are part of the process of 57. A. NO CHANGE
57 B. share the struggles
all work, I made my editors worry and at the same time reinforced
C. shared the struggles
all those stereotypes about women who can’t stand the pressure. D. will share the struggle

Questions 58–60 ask about the passage as a whole.


58. For the sake of unity and coherence, Paragraph 2 should
be placed:
F. where it is now.
G. before Paragraph 1.
H. after Paragraph 4.
J. after Paragraph 5.

59. The writer is considering eliminating Paragraph 4. If the


writer removed this paragraph, the essay would
primarily lose:
A. relevant details about the mistakes the writer made
that led to her ultimate realization.
B. historical information regarding women in the
workplace.
C. relevant details regarding the writer’s male
colleagues’ behavior.
D. a revealing anecdote about the writer’s experience
with her friends.

CONTINUE
  829
Peterson's ACT® Prep Guide 2018
Practice Test 3

60. Which of the following assignments would this essay


most clearly fulfill?
F. Write a persuasive essay about the benefits of
holding a job.
G. Write an essay comparing current and past
business environments for women.
H. Write an essay about a lesson you learned from a
professional experience.
J. Write an essay about an experience in which your
personal integrity was challenged.

PASSAGE V

Edwidge Danticat, A Born Writer

[1]

Those who live in countries where a large proportion of the

population is illegible share their stories orally. In Haiti, 61. A. NO CHANGE


61 B. are ineligible
C. is illiterate
D. are illiterate

one of the world’s smaller countries, when someone has a tale 62. F. NO CHANGE
62 G. a small country
H. a paticularly small country
J. DELETE the underlined portion

to tell, he or she will call out Krik? Neighbors: friends, and relatives 63. A. NO CHANGE
63 B. Neighbors;
C. Neighbors
D. Neighbors,

will then gather around with an answering call of Krak!, signaling

there willingness to listen. 64. F. NO CHANGE


64 G. they’re
H. their
J. they are

  830
www.petersons.com
Practice Test 3

[2]

The Haitian-born writer Edwidge Danticat would have been 65. A. NO CHANGE
65 B. having been
only twenty-six when she took these two words and made them
C. was being
the title for her collection of stories. The nine stories in Krik?Krak! D. was

focus on the hardships of living under a dictatorship. 66 66. For the sake of the logic and coherence of this
paragraph, Sentence 2 should be placed:
F. where it is now.
G. before Sentence 1.
H. after Sentence 3.
J. after Sentence 4.

Families are forced to flee Haiti and seek new lives, in the United 67. A. NO CHANGE
67 B. lives’ in
States.
C. lives in
D. life in
The book received much critical acclaim and even became a 68. F. NO CHANGE
68
G. many critically
finalist for the National Book Award.
H. too critical
[3] J. much criticism

Born in Port-au-Prince in 1969, her family moved to New York 69. A. NO CHANGE
69
B. Danticat moved to New York City when she was
City when Danticat was twelve. She spoke little as a new
69 twelve
immigrant, because when she did speak, other children made C. New York City is where Danticat moved when she
was twelve
D. when she was twelve Danticat moved to New
York City

fun of her heavily accented English. 70 Her thesis in graduate 70. Which of the following sentences, if added here,
would best provide a transition from the description
school later became the novel Breath,Eyes, Memory. That
of Danticat as a young girl to that of Danticat as
an author?
F. Danticat refrained from criticizing them in return,
however, and was successful in the end.
G. Most Haitians speak Creole, and the language is
quite different from American English.
H. Many writers have had difficult childhoods.
J. Although she was silent much of the time,
Danticat watched and remembered, as if already
thinking like a writer.

CONTINUE
  831
Peterson's ACT® Prep Guide 2018
Practice Test 3

novel, which was subsequently chosen by Oprah Winfrey for

her book club, featured a heroine who, like the author, moved 71. A. NO CHANGE
71 B. not unlike the author
from Haiti to New York City at the age of twelve. Danticat’s
C. just as the author did
third book, The Farming of Bones, is also set in Haiti. D. in much the same manner as the author

[4]

This young authors chosen subject matter, as well as the 72. F. NO CHANGE
72 G. authors’
Creole-accented language she uses to tell her stories, demonstrate
H. author’s
that while Danticat has left Haiti for her adopted country of J. author

America, she hasn’t forgotten both the land of her birth and its 73. A. NO CHANGE
73 B. either the land of her birth or its
brave people.
C. neither the land of her birth or its
D. neither the land of her birth nor its

Questions 74 and 75 ask about the passage as a whole.


74. The writer wishes to open the essay with a sentence that
will set the theme and tone of the essay. Which of the
following would most effectively accomplish this?
F. Whether or not they can read, people all over the
world love stories.
G. One of my favorite books is a collection of stories
set in Haiti.
H. The problem of illiteracy results in a variety of
consequences for people all over the world.
J. Have you ever wondered what it feels like not to be
able to read?

75. Suppose the writer’s goal had been to provide a detailed


history of the hardships the people of Haiti have
endured. Would this essay accomplish that goal?
A. Yes, because the essay mentions that the plot of
Krik? Krak! involves such hardships.
B. Yes, because the essay is about Edwidge Danticat,
who fled Haiti because of the hardships she
endured while living there.
C. No, because the essay does not provide nearly
enough information about Haiti to serve as a
detailed history.
D. No, because the essay does not imply that the
people of Haiti ever suffered hardships.

STOP
  832
www.petersons.com
Practice Test 3

MATHEMATICS TEST
60 Minutes—60 Questions

DIRECTIONS: For each of the following items, solve each problem, choose the correct answer, and then fill in the corre-
sponding circle on the answer sheet. If you encounter problems that take too much time to solve, move on. Solve as many
problems as you can; then return to the others in the time remaining for the test.

You may use a calculator on this test for any problems you choose, but some of the problems may best be solved without
the use of a calculator.

SHOW YOUR WORK HERE

Practice Test 3 — mathematics


Note: Unless otherwise stated, assume the following:

1.   Illustrative figures are NOT necessarily drawn


to scale.
2.   Geometric figures lie in a plane.
3.   The word line indicates a straight line.
4.   The word average indicates arithmetic mean.

1. Of the 80 children at a daycare center, 32 missed at least


1 day because of illness in January. What percent of the
children at the center missed at least 1 day due to illness
in January?
A. 4%
B. 16%
C. 24%
D. 32%
E. 40%

2. Three lines intersect to form the triangle below. What is


the value of x?

40º
55º

F. 55º
G. 75º
H. 85º
J. 95º
K. 105º

CONTINUE
  833
Peterson's ACT® Prep Guide 2018
Practice Test 3

3. A businessperson calculates that it costs her p dollars SHOW YOUR WORK HERE
per hour in parts costs and l dollars per hour in labor
costs to operate a certain piece of machinery. Which of
the following expressions would provide her with the
total cost of operating this machine for x hours?
A. plx
B. (px)(lx)
C. x(p + l)
D. pl
E. p + l

4. Which of the following is the graph of j(x) = 3x – 2?


J. y
F. y

x
(–2,1)
–1 2
x

K.
G. y y

x
–2 –1

x
–1

–2

H.
y

–1

–2

  834
www.petersons.com
Practice Test 3

5. A phone company charges $1 for a calling card, plus 25 SHOW YOUR WORK HERE
cents per minute for the length of a call. If Jacob uses a
calling card to make 7 calls, each of which lasts 3
minutes, what is his total cost for the calls?
A. $1.75
B. $2.75
C. $5.25
D. $5.50
E. $6.25

6. An amateur artist bought a painting at a yard sale for


$85. She restored the painting and sold it via an online
auction for $125. What percent profit did she earn if she
had to pay 3% in fees to the manager of the auction site?

36.25
F. × 100%
85
G. 36.25 × 100%
125
H. 40%

J. 40
× 100%
85
K. 36.25%

7. Which of the following is equal to the sum of


2 1
, , and 0.177 ?
5 8

A. 0.525
B. 0.577
C. 0.602
D. 0.692
E. 0.702

8. In the figure below, BC = 2 and BD = 10. If the area of


triangle ABC is 5 square inches, what is the area, in
square inches, of triangle ACD?

B C D

F. 8
G. 15
H. 20
J. 25
K. 30

CONTINUE
  835
Peterson's ACT® Prep Guide 2018
Practice Test 3

9. If w =
2 1
and z = − , what is the value of
w+z
?
SHOW YOUR WORK HERE
3 9 2(w − z )

1
A. −
3
5
B. −
14
5
C.
14
1
D.
2
14
E.
5

10. What is the asymptote for the graph of


F(x) = 5 – log(x + 4)?
F. x = –4
G. y=5
H. y = –4
J. x=4
K. x=5
2x
11. What is the inverse of the function g( x ) = ?
x −1
x −1
A. g −1( x ) =
2x
1
B. g −1( x ) =
x −2
x −2
C. g −1( x ) =
x
x
D. g −1( x ) =
x −2
x ( x − 1)
E. g −1( x ) =
2

12. Suppose p, q, and r are prime numbers. What is the least


common multiple of the whole numbers p2qr3, pq3r3,
and p3r4?
F. pqr3
G. p2q3r3
H. p3q3r4
J. pqr
K. p6q4r10

  836
www.petersons.com
Practice Test 3

13. Colin earns an annual salary of C dollars, and Daryl earns SHOW YOUR WORK HERE
an annual salary of D dollars. If the difference between
Colin’s and Daryl’s average monthly salary is $500, which
of the following represents the relationship between C
and D?

A. C − D = 500

B. C + D = 500

C +D
C. = 6 , 000
2
D. C − D = 6 , 000

E. C + D = 6 , 000

14. A 1980s-style arcade charges customers an admission


fee of $8.50, and it costs just $0.25 per game. Mike and
Stan, together, spent $35. If Stan played three fewer
than twice the number of games Mike played, how
many games did they play, total?
F. 18
G. 25
H. 47
J. 72
K. 75

15. In the figure below, segments AD and BC intersect at


point E and AB  CD. What is the measure of ∠CED ?

D
B
40º
60º

A
C

A. 40°
B. 60°
C. 80°
D. 100°
E. Cannot be determined from the given information

CONTINUE
  837
Peterson's ACT® Prep Guide 2018
Practice Test 3

16. Two eight-sided dice are used in a board game. One is SHOW YOUR WORK HERE
red with faces labeled as A, B, C, D, E, F, G, and H, while
the other is blue with faces labeled as 0, 10, 20, 50, 100,
500, 1,000, and 5,000. If the result of rolling both dice is
recorded, how many possible outcomes are there?
F. 8
G. 16
H. 32
J. 64
K. 128

17. What is the slope of a line perpendicular to the line


whose equation is 2x – 5y – 1 = 0?
5
A. −
2
1
B. −
2
2
C.
5
1
D.
2

E. 2

18. A painter calculates that one gallon of paint can cover a


400 square foot wall with no paint left over. A house has
four walls that are 20 feet tall. Two of the walls are 30
feet long, and two of the walls are 40 feet long. How
many gallons of paint will the painter need to cover all
four walls?
1
F. 3
2

G. 1
5
2
H. 6
J. 7
K. 30

19. What value of w makes the proportion true?


2w + 1 5
=
3 3 6
A.
4
4
B.
3
7
C.
6
D. 9
E. 15

  838
www.petersons.com
Practice Test 3

20. In the figure below, triangles BCA and DEF are similar. SHOW YOUR WORK HERE
What is the length of segment EF?

B F

16
40
30 D
24

A 20 C E

F. 28
G. 30
H. 32
J. 36
K. 40

21. For all nonnegative values of x and y, to which


43 x 7 y
expression is equal?
32 x 4 y 3

8 14 23
A. x y
3
2
4x3
B.
3y
3
8x 2
C.
3y

8 21 2
D. x y
3

3
8x
E. 2
3y

22. The average of six test scores is 80. When two more tests
are included, the average for all eight tests is 85. What is
the average score of the two added tests?
F. 70
G. 85
H. 90
J. 95
K. 100

CONTINUE
  839
Peterson's ACT® Prep Guide 2018
Practice Test 3

23. What is the area of right triangle ABC below, in square SHOW YOUR WORK HERE
centimeters?

15 cm
9 cm

A C

A. 12
B. 36
C. 54
D. 108
E. 135

24. It took Renee exactly 4 minutes and 26 seconds to


download a file from the Internet to her computer. At
that same rate, how long would it take Renee to
1
download a file that is 2 times as large as the first file?
2

F. 10 minutes and 5 seconds


G. 10 minutes and 26 seconds
H. 10 minutes and 52 seconds
J. 11 minutes and 5 seconds
K. 11 minutes and 15 seconds

25. A college campus’s central green is a square lawn


bordered by footpaths 100 feet long on each side. The
college plans to create a diagonal footpath that would
connect the southwest corner of the green to the
northeast corner of the green. Approximately how many
feet shorter would the new path be than the shortest
possible route on the existing footpaths?
A. 40
B. 50
C. 60
D. 140
E. 150

26. Nick sold his riding mower for $5,400. If this is 30% less
than what he originally paid, how much did the mower
originally cost?
5, 400
F. dollars
0.7
G. 0.7(5,400) dollars
H. 0.3(5,400) dollars
J. 5,400 + 0.7 dollars
K. 5, 400 dollars
0.3

  840
www.petersons.com
Practice Test 3

27. Ben has won the bid to repaint the local high school SHOW YOUR WORK HERE
football stadium. If one gallon of paint covers an area of
150 square feet, what is the minimum number of gallons
he will need to paint two rectangular walls that are 100
yards long and 15 yards high?
A. 20
B. 60
C. 90
D. 150
E. 180

28. In the figure below, WXYZ is a square. Points are chosen


on each pair of adjacent sides of WXYZ to form four
congruent right triangles, as shown below. Each of the
triangles has one leg that is three times longer than the
other leg. What is the ratio of the area of the four
triangles to the area of the shaded region?

X Y

3a

W a Z

3
F.
8
2
G.
5

1
H.
2
3
J.
5

3
K.
4

CONTINUE
  841
Peterson's ACT® Prep Guide 2018
Practice Test 3

1

1 SHOW YOUR WORK HERE
29. 10 100 =
1 1

5 25

9
A.
625
1
B.
4
9
C.
16

16
D.
9

E. 625
9

30. What is the equation of the linear function whose graph


is below?

(–2,–1)

(4,–3)

F. f(x) = –3x – 7
1 5
G. f ( x ) = − x −
3 3
H. f(x) = –3x – 2
1
J. f (x) = − x − 2
3
1 5
K. f ( x ) = x +
3 3

  842
www.petersons.com
Practice Test 3

31. At a township meeting, thirty residents are asked how SHOW YOUR WORK HERE
many robocalls they received, on average, in a typical
evening during election season. They responded as
follows:

5 1 0 8 5 4
2 4 1 8 0 5
2 1 0 0 7 8
1 5 1 2 7 1
1 4 4 1 3 1

What is the relationship between the mean, median,


and mode of this data set?

A. mean < median < mode


B. mean < mode < median
C. mode < median < mean
D. median < mean < mode
E. mode < mean < median

2
32. For 90 < q < 180 , if sinq = , then cotq =
3
5
F. −
2
G. −2 5

5
H. −
3

5
J.
3
2 5
K. −
9

CONTINUE
  843
Peterson's ACT® Prep Guide 2018
Practice Test 3

33. The average of x numbers is 15. If two of the numbers SHOW YOUR WORK HERE
are each increased by y, the new average will be
increased by how much?
A. 2y
B. y
x
C.
2y

D. y
x

2y
E.
x

34. Points A, B, and C below lie on the circumference of a


circle with center O. If the unshaded sectors AOB and
2 1
BOC comprise and of the area of the circle,
5 3
respectively, what is the measure of ∠AOC ?
B

C
A
F. 48°
G. 90°
H. 92°
J. 96°
K. 264°

35. A 12-foot ladder is propped against a 10-foot house as


shown below. Approximately how many feet is the base
of the ladder from the wall of the house?
t

10 feet
e
fe
12

A. 2.0
B. 4.0
C. 6.6
D. 15.6
E. 44

  844
www.petersons.com
Practice Test 3

36. What is the area of a circle with a diameter of 12 yards? SHOW YOUR WORK HERE
F. 36π square yards
2

G. 144 square yards


H. 12π square yards
J. 36π square yards
K. 144π square yards

37. There are 3 teaspoons in 1 tablespoon. A bottle of cough


medicine contains 24 tablespoon-sized servings and
costs $4.50. What is the approximate cost per teaspoon?
A. $0.06
B. $0.15
C. $0.19
D. $0.60
E. $1.50

38. A line passing through the point (–1, 4) is perpendicular


to the line passing through (–3, 2) and (–3, –5). What is
the equation of this line?
F. y = x + 5
G. y = 4
H. 9x + 2y = 5
J. x = –1
K. y = –x – 5

39. A right triangle has hypotenuse with length 5 yards and


one leg has length 2 yards. What is the area of this
triangle?
A. 3 square yards
B. 29 square yards
C. 5 square yards
D. 21 square yards
5
E. 21 square yards
2

CONTINUE
  845
Peterson's ACT® Prep Guide 2018
Practice Test 3

2  4 SHOW YOUR WORK HERE


40. Let h( x ) = 1− . Compute h  −  .
3x  x
F. x
x + 24
G.
x
2x +1
H.
12
J. 8 − 12 x
3x 2
K. x+6
6
41. What is the y-intercept, if one exists, of the following
piecewise-defined function?

 x + 1 + 5, x ≤ −2

F( x ) =  4 + 3x , − 2 < x ≤ 1

 − x + 1, x >1
2

A. No y-intercept

B. 4

3
C. 1
D. 4
E. 6

42. What is the least possible value for the product xy,
if x + y = 24 and x and y are both prime numbers?
F. 23
G. 44
H. 95
J. 119
K. 143

43. The center of a circle is (–4, 3). If the point (2, –2) lies on
this circle, what is the equation of this circle?

A. (x – 4)2 + (y + 3)3 = 61
B. (x – 4)2 + (y + 3)3 = 61
C. (x + 4)2 + (y – 3)3 = 11
D. (x – 2)2 + (y + 2)2 = 61
E. (x + 4)2 + (y – 3)2 = 61

44. If 0.3z – 0.24 = 1.2(0.3 – z), what is the value of z i 10 −3?


F. 0.0004
G. 0.004
H. 0.4
J. 400
K. 4,000

  846
www.petersons.com
Practice Test 3

45. In the isosceles right triangle below, what is the value of SHOW YOUR WORK HERE
tan x°?

2
A.
2

B. 1

C. 2

3
D.
2

E. 3

46. What is the area of the rectangle shown below in the


standard (x, y) coordinate plane?

(7,2)
x
(–5,–2)

(–1,–6)

F. 32 2
G. 64
H. 24 2
J. 128
K. 144

CONTINUE
  847
Peterson's ACT® Prep Guide 2018
Practice Test 3

3 2
SHOW YOUR WORK HERE
47. If x y is positive, then which of the following could
z
be true?
I. x and z are both negative
II. x is negative and z is positive
III. x is zero

A. I only
B. II only
C. I and II
D. II and III
E. I, II, and III
 1 
48. What is the value of log5  ?
 625 
F. –4
G. –3
1
H.
4
J. 4
K. 125

49. If x is inversely related to y and y is directly related to z,


which of the following expressions gives one possibility
for y in terms of x and z?
A. x
y=
z
B. z
y=
x
C. y = zx
D. y = xz
E. y = zx

50. Mary is weatherproofing her rain gutters for the winter.


To prevent freezing, she needs to install heat tape along
the gutters. The local hardware store charges $8.10 per
3-foot segment. What is the cost for 114 feet of heat
tape if 5% sales tax is added to the cost?
F. $292.41
G. $307.80
H. $323.19
J. $923.40
K. $969.57

  848
www.petersons.com
Practice Test 3

3(1− 2 y ) 2 − 4 y SHOW YOUR WORK HERE


51. If − = y , what is y–1?
4 3
A. –16
1
B. − 5

C. 1

16
D. 5
E. 16
Ê pˆ
52. What is the period of the graph of y = 4 cos Á 3 x - ˜ + 1?
Ë 2¯
p
F.
6
2p
G.
3
H. 3
J. 4
K. 2π

53. The length of a diagonal of a square is 8 inches long.


What is the perimeter of the square?
A. 4 2 inches
B. 8 2 inches
C. 16
D. 16 2 inches
E. 32 inches

54. A positive number x is increased by 20 percent, and the


result is then decreased by 30 percent. The final result is
equal to which of the following?
F. x decreased by 50 percent
G. x decreased by 16 percent
H. x decreased by 10 percent
J. x increased by 10 percent
K. x increased by 24 percent

55. A 50-foot wire is attached to the top of an electric pole


and is anchored on the ground. If the wire rises in a
straight line at a 70° angle from the ground, how many
feet tall is the pole?
A. 50 sin 70°
B. 50 cos 70°
C. 50 tan 70°
cos 70°
D.
50

E. 50
cos 70°

CONTINUE
  849
Peterson's ACT® Prep Guide 2018
Practice Test 3

56. What is the complete solution set of the inequality SHOW YOUR WORK HERE
2|3x + 1| > 8?
F. {x: x > 5}
5
G. {x: x < − }
3
5
H. {x: x > 1 or x < − }
3
17
J. {x: x > 5 or x < − }
5
K. {x: x > 1}

57. For what values of c do the lines 2x – cy = 1 and


8cx –2 = y never intersect in the standard xy-plane?

A. c = 0
1 1
B. c=− ,c=
2 2
C. c = –2, c = 2
D. c = –4, c = 4
1 1
E. c=− ,c=
16 16
58. Lines m and n are perpendicular in the standard (x, y)
coordinate plane below. Which of the following is the
equation for line n?
y

n (–1,5)

(–2,2)
x

F. y = –x + 5
G. y = –2x + 3
H. 2y = –x – 4
J. 3y = –x + 14
K. 3y = x + 16

59. Equally sized cards numbered from 1 to 30 are placed in


a box. If one card is selected at random, what is the
probability that the number on the card is either odd or
divisible by 5?
A. 0.18
B. 0.40
C. 0.50
D. 0.60
E. 0.70

  850
www.petersons.com
Practice Test 3

60. While shopping at a clothing store, Ben finds that a shirt SHOW YOUR WORK HERE
and two ties cost $105, while two shirts and one tie cost
$135. If the store charges the same price for all of its
shirts and the same price for all of its ties, what is Ben’s
cost if he wants to buy just one tie?
F. $15
G. $25
H. $30
J. $50
K. $55

STOP
  851
Peterson's ACT® Prep Guide 2018
Practice Test 3

READING TEST
35 Minutes—40 Questions

DIRECTIONS: There are four passages in this test. Each passage is followed by several questions. Read each passage, select
the best answer to each related question, and fill in the corresponding circle on the answer sheet. You may look back at the
passages as often as you need.

PASSAGE I “Aye, so it is,” cried her mother, “and Mrs. Long does not
come back till the day before; so it will be impossible for her
PROSE FICTION: Passage A is an excerpt from Pride and Prejudice
30 to introduce him, for she will not know him herself.”
by Jane Austen. Passage B is an excerpt from Middlemarch by
George Eliot. “Then, my dear, you may have the advantage of your friend,

Practice Test 3 — reading


and introduce Mr. Bingley to her.”
Passage A
by Jane Austen “Impossible, Mr. Bennet, impossible, when I am not
acquainted with him myself; how can you be so teasing?”
Mr. Bennet was among the earliest of those who waited on
Mr. Bingley. He had always intended to visit him, though to the 35 “I honour your circumspection. A fortnight’s acquaintance
last always assuring his wife that he should not go; and till the is certainly very little. One cannot know what a man really is
evening after the visit was paid she had no knowledge of it. It by the end of a fortnight. But if we do not venture somebody
5 was then disclosed in the following manner. Observing his else will; and after all, Mrs. Long and her daughters must stand
second daughter employed in trimming a hat, he suddenly their chance; and, therefore, as she will think it an act of
addressed her with: 40 kindness, if you decline the office, I will take it on myself.”

“I hope Mr. Bingley will like it, Lizzy.” The girls stared at their father. Mrs. Bennet said only,
“Nonsense, nonsense!”
“We are not in a way to know what Mr. Bingley likes,” said her
10 mother resentfully, “since we are not to visit.” “What can be the meaning of that emphatic exclamation?”
cried he. “Do you consider the forms of introduction, and the
“But you forget, mamma,” said Elizabeth, “that we shall meet
45 stress that is laid on them, as nonsense? I cannot quite agree
him at the assemblies, and that Mrs. Long promised to introduce
with you there. What say you, Mary? For you are a young lady
him.”
of deep reflection, I know, and read great books and make
“I do not believe Mrs. Long will do any such thing. She has extracts.”
15 two nieces of her own. She is a selfish, hypocritical woman, and
Mary wished to say something sensible, but knew not how.
I have no opinion of her.”
Passage B
“No more have I,” said Mr. Bennet; “and I am glad to find that
by George Eliot
you do not depend on her serving you.”
Celia thought privately, “Dorothea quite despises Sir James
Mrs. Bennet deigned not to make any reply, but, unable to
Chettam; I believe she would not accept him.” Celia felt that
20 contain herself, began scolding one of her daughters.
this was a pity. She had never been deceived as to the object
“Don’t keep coughing so, Kitty, for Heaven’s sake! Have a little of the baronet’s interest. Sometimes, indeed, she had reflected
compassion on my nerves. You tear them to pieces.” 5 that Dodo would perhaps not make a husband happy who
had not her way of looking at things; and stifled in the depths
“Kitty has no discretion in her coughs,” said her father; “she
of her heart was the feeling that her sister was too religious
times them ill.”
for family comfort. Notions and scruples were like spilt needles,
25 “I do not cough for my own amusement,” replied Kitty fretfully. making one afraid of treading, or sitting down, or even eating.
“When is your next ball to be, Lizzy?”
10 When Miss Brooke was at the tea-table, Sir James came to
“Tomorrow fortnight.” sit down by her, not having felt her mode of answering him

CONTINUE
  853
Peterson's ACT® Prep Guide 2018
Practice Test 3

at all offensive. Why should he? He thought it probable that Questions 1–3 ask about Passage A.
Miss Brooke liked him, and manners must be very marked indeed
1. Lizzy is described in the passage as the Bennets’:
before they cease to be interpreted by preconceptions either
15 confident or distrustful. She was thoroughly charming to him, A. youngest daughter.
but of course he theorized a little about his attachment. He was B. oldest daughter.
made of excellent human dough, and had the rare merit of C. sensible daughter.
knowing that his talents, even if let loose, would not set the D. second daughter.
smallest stream in the county on fire: hence he liked the prospect 2. As it is used in line 22, the phrase “You tear them to
20 of a wife to whom he could say, “What shall we do?” about this pieces” most likely means:
or that; who could help her husband out with reasons, and
would also have the property qualification for doing so. F. “you are extremely destructive.”
G. “you have a tendency toward violence.”
As to the excessive religiousness alleged against Miss Brooke, H. “you are making me upset.”
he had a very indefinite notion of what it consisted in, and J. “you are making me physically ill.”
25 thought that it would die out with marriage. In short, he felt
himself to be in love in the right place, and was ready to endure 3. When Mr. Bennet tells his wife that she should
a great deal of predominance, which, after all, a man could introduce Mr. Bingley to Lizzy, she feels that Mr. Bennet:
always put down when he liked. Sir James had no idea that he A. has lost his mind.
should ever like to put down the predominance of this handsome B. is breaching etiquette.
30 girl, in whose cleverness he delighted. Why not? A man’s mind— C. does not understand his wife.
what there is of it—has always the advantage of being mas- D. does not love his wife.
culine,—as the smallest birch-tree is of a higher kind than the
Questions 4–6 ask about Passage B.
most soaring palm,—and even his ignorance is of a sounder
quality. Sir James might not have originated this estimate; but 4. According to the passage, people seem to think that
35 a kind Providence furnishes the limpest personality with a little Miss Brooke is:
gunk or starch in the form of tradition. F. too rude.
“Let me hope that you will rescind that resolution about the G. too religious.
horse, Miss Brooke,” said the persevering admirer. “I assure you, H. too happy.
riding is the most healthy of exercises.” J. too unscrupulous.

40 “I am aware of it,” said Dorothea, coldly. “I think it would do 5. In the context of the passage, the primary function of
Celia good—if she would take to it.” lines 28–30 is to:

“But you are such a perfect horsewoman.” A. quote something Sir James said about Miss
Brooke.
“Excuse me; I have had very little practice, and I should be B. reveal Sir James’s true feelings about Miss Brooke.
easily thrown.” C. contradict the idea that Sir James was charming.
45 “Then that is a reason for more practice. Every lady ought to D. list examples of why Sir James loved Miss Brooke.
be a perfect horsewoman, that she may accompany her 6. As it is used in line 3, the word pity most nearly means:
husband.”
F. sorry.
“You see how widely we differ, Sir James. I have made up my G. compassion.
mind that I ought not to be a perfect horsewoman, and so I H. sympathy.
50 should never correspond to your pattern of a lady.” Dorothea J. shame.
looked straight before her, and spoke with cold brusquerie, very
Questions 7–10 ask about both passages.
much with the air of a handsome boy, in amusing contrast with
the solicitous amiability of her admirer. 7. Given the way they are presented in each passage, Mrs.
Bennet and Miss Brooke are best described as:
A. demure.
B. disgusted.
C. short-tempered.
D. amusing.

  854
www.petersons.com
Practice Test 3

8. In what way is Kitty in Passage A DIFFERENT from Miss There had also been heartening signs in her own country,
Brooke in Passage B? 20 which she hoped were the preparation for more successful

F. Kitty is concerned about how others interpret her militancy to come. She had exulted in The Revolution in 1868
behavior. over the attempt of women to vote in Vineland, New Jersey.
G. Kitty does not care about what is expected of Encouraged by the enfranchisement of women in Wyoming in
women. 1869, Mary Olney Brown and Charlotte Olney French had cast
H. Kitty uses hostility to mask her romantic feelings. 25 their votes in Washington Territory. A young widow, Marilla
J. Kitty is extremely selfish and hypocritical. Ricker, had registered and voted in New Hampshire in 1870,
claiming this right as a property holder, but her vote was refused.
9. The narrators of Passage A and Passage B reveal the In 1871, Nannette B. Gardner and Catherine Stebbins in Detroit,
inner thoughts of which characters from each passage? Catherine V. White in Illinois, Ellen R. Van Valkenburg in Santa
A. Kitty and Celia 30 Cruz, California, and Carrie S. Burnham in Philadelphia registered
B. Elizabeth and Miss Brooke and attempted to vote. Only Mrs. Gardner’s vote was accepted.
C. Mr. Bingley and Sir James That same year, Sarah Andrews Spencer, Sarah E. Webster, and
D. Mary and Celia seventy other women marched to the polls to register and vote
in the District of Columbia. Their ballots refused, they brought
10. It can be inferred that Mr. Bingley in Passage A and Sir
35 suit against the Board of Election Inspectors, carrying the case
James in Passage B are both:
unsuccessfully to the Supreme Court of the United States.
F. extremely charming. Another test case based on the Fourteenth Amendment had
G. uninterested in etiquette. also been carried to the Supreme Court by Myra Bradwell, one
H. unmarried men. of the first women lawyers, who had been denied admission to
J. in love with the women they meet. 40 the Illinois bar because she was a woman.

PASSAGE II
With the spotlight turned on the Fourteenth Amendment by
SOCIAL SCIENCE: The following passage is excerpted from a these women, lawyers here and there throughout the country
biography of social reformer and activist Susan B. Anthony, who were discussing the legal points involved, many admitting that
worked to ensure that women have the right to vote in the women had a good case. Even the press was friendly.
United States.
45 Susan had looked forward to claiming her rights under the
Susan preached militancy to women throughout the presi- Fourteenth and Fifteenth Amendments and was ready to act.
dential campaign of 1872, urging them to claim their rights She had spent the thirty days required of voters in Rochester
under the Fourteenth and Fifteenth Amendments by registering with her family and as she glanced through the morning paper
and voting in every state in the Union. of November 1, 1872, she read these challenging words, “Now
50 Register … If you were not permitted to vote you would fight
5 Even before Francis Minor had called her attention to the for the right, undergo all privations for it, face death for it… .”
possibilities offered by these amendments, she had followed
with great interest a similar effort by Englishwomen who, in This was all the reminder she needed. She would fight for
1867 and 1868, had attempted to prove that the “ancient legal this right. She put on her bonnet and coat, telling her three
rights of females” were still valid and entitled women property sisters what she intended to do, asked them to join her, and
10 holders to vote for representatives in Parliament, and who 55 with them walked briskly to the barber shop where the voters
claimed that the word “man” in Parliamentary statutes should of her ward were registering. Boldly entering this stronghold
be interpreted to include women. In the case of the 5,346 of men, she asked to be registered. The inspector in charge,
householders of Manchester, the court held that “every woman Beverly W. Jones, tried to convince her that this was impossible
is personally incapable” in a legal sense. This legal contest had under the laws of New York. She told him she claimed her right
15 been fully reported in The Revolution, and disappointing as the 60 to vote not under the New York constitution but under the
verdict was, Susan looked upon this attempt to establish justice Fourteenth Amendment, and she read him its pertinent lines.
as an indication of a great awakening and uprising among Other election inspectors now joined in the argument, but she
women. persisted until two of them, Beverly W. Jones and Edwin F. Marsh,
both Republicans, finally consented to register the four women.

CONTINUE
  855
Peterson's ACT® Prep Guide 2018
Practice Test 3

65 This mission accomplished, Susan rounded up twelve more 12. Based on information in the passage, Susan B. Anthony
women willing to register. The evening papers spread the sen- would most likely promote which of the following
sational news, and by the end of the registration period, fifty methods to secure one’s civil rights?
Rochester women had joined the ranks of the militants. F. Conduct a violent revolution.
G. Complain to the press.
On election day, November 5, 1872, Susan gleefully wrote
H. Vote for sympathetic politicians.
70 Elizabeth Stanton, “Well, I have gone and done it!!—positively
J. Invoke the U.S. constitution.
voted the Republican ticket—Strait—this a.m. at 7 o’clock—&
swore my vote in at that.... All my three sisters voted—Rhoda 13. The efforts described in the third paragraph are
deGarmo too—Amy Post was rejected & she will immediately presented in order to illustrate the idea that:
bring action against the registrars.... Not a jeer not a word—not A. a militant effort to secure women the right to vote
75 a look—disrespectful has met a single woman.... I hope the was in full force.
mornings telegrams will tell of many women all over the country B. Susan B. Anthony was not alone in her efforts to
trying to vote.... I hope you voted too.” secure women the right to vote.
C. every woman in the United States was angered
Election day did not bring the general uprising of women for that they did not have the right to vote.
which Susan had hoped. In Michigan, Missouri, Ohio, and D. activists would use the Supreme Court as their
80 Connecticut, as in Rochester, a few women tried to vote. In New main tool for securing equal rights.
York City, Lillie Devereux Blake and in Fayetteville, New York,
Matilda Joslyn Gage had courageously gone to the polls only 14. The author of the passage appears to feel that the
to be turned away. Elizabeth Stanton did not vote on November question “What does it take to enact social change?” is
5, 1872, and her lack of enthusiasm about a test case in the best answered by the assertion that it takes:
85 courts was very disappointing to Susan. F. the dedicated work of a single visionary.
G. an effort to vote out the politicians currently in
However, the fact that Susan B. Anthony had voted won charge.
immediate response from the press in all parts of the country. H. a unified effort by many dedicated individuals.
Newspapers in general were friendly, the New York Times boldly J. a lawsuit brought in front of the US Supreme
declaring, “The act of Susan B. Anthony should have a place in Court.
90 history,” and the Chicago Tribune venturing to suggest that she
15. The author states in paragraph 6 that Beverly W. Jones
ought to hold public office. The cartoonists, however, reveling
and Edwin F. Marsh “finally consented to register the
in a new and tempting subject, caricatured her unmercifully,
four women” because:
the New York Graphic setting the tone. Some Democratic papers
condemned her, following the line of the Rochester Union and A. Susan B. Anthony’s argument was so persistent.
95 Advertiser which flaunted the headline, “Female Lawlessness,” B. Susan B. Anthony read lines from the Fourteenth
and declared that Miss Anthony’s lawlessness had proved Amendment.
women unfit for the ballot. C. Susan B. Anthony turned election inspectors
against each other.
11. The passage specifies that when more than seventy D. Susan B. Anthony rounded up twelve more women
women were refused the right to vote in the District of willing to register.
Columbia in 1871 they:
16. As it is used in line 61, the word pertinent most nearly
A. tried to vote anyway, but only the vote of means:
Nannette B. Gardner was accepted.
F. relevant.
B. sued the Board of Election Inspectors in a case
G. interesting.
heard by the US Supreme Court.
H. persistent.
C. organized a march on Washington, D.C., to protest
J. remorseful.
such unfair treatment.
D. traveled to Washington Territory to vote there
instead.

  856
www.petersons.com
Practice Test 3

17. According to the passage, the news media reported on 10 ten or twenty francs at the most. Today they sell for considerable
the registration of women voters in Rochester after: prices, although the government has not yet acquired any work
A. fifty women had joined the ranks of the militants. by Monticelli for the public galleries. The mysterious power
B. Anthony wrote a gleeful letter to Elizabeth alone of these paintings secures him a fame which is, alas!
Stanton. posthumous. Many Monticellis have been sold by dealers as
15 Diaz’s; now they are more eagerly looked for than Diaz, and
C. Amy Post sued the registrars who would not allow
her to vote. collectors have made fortunes with these small canvases bought
D. Anthony had successfully gotten sixteen women formerly, to use a colloquial expression which is here only too
to register to vote. literally true, “for a piece of bread.”

18. In the third paragraph (lines 19-40), the author Monticelli painted landscapes, romantic scenes, “fêtes
discusses Myra Bradwell in a manner that: 20 galantes” in the spirit of Watteau, and still-life pictures: one could
F. emphasizes how Myra Bradwell was not qualified not imagine a more inspired sense of color than shown by these
for the Illinois bar. works which seem to be painted with crushed jewels, with
G. exaggerates the difficulties of being a woman powerful harmony, and beyond all with an unheard-of delicacy
during Susan B. Anthony’s time. in the perception of fine shades. There are tones which nobody
H. highlights the challenges women faced during 25 had ever invented yet, a richness, a profusion, a subtlety which
Susan B. Anthony’s time. almost vie with the resources of music. The fairyland atmosphere
J. showcases the brightest women who lived during of these works surrounds a very firm design of charming style,
Susan B. Anthony’s time. but, to use the words of the artist himself, “in these canvases
the objects are the decoration, the touches are the scales, and
19. According to the ninth paragraph, Susan B. Anthony the light is the tenor.” Monticelli has created for himself an
30
was NOT criticized by: entirely personal technique which can only be compared with
A. the New York Graphic. that of Turner; he painted with a brush so full, fat and rich, that
B. the Rochester Union and Advertiser. some of the details are often truly modeled in relief, in a sub-
C. many editorial cartoonists. stance as precious as enamels, jewels, ceramics—a substance
D. the Chicago Tribune. 35 which is a delight in itself. Every picture by Monticelli provokes
astonishment; constructed upon one color as upon a musical
20. One of the main points made in the eighth paragraph
theme, it rises to intensities, which one would have thought
is that although Anthony brought a great deal of
impossible. His pictures are magnificent bouquets, bursts of
attention to the movement to win women the right to
joy and color, where nothing is ever crude, and where everything
vote in the United States, her initial efforts:
40 is ruled by a supreme sense of harmony.
F. inspired women such as Lillie Devereux Blake and
Matilda Joslyn Gage to vote. Claude Lorrain, Watteau, Turner and Monticelli constitute
G. were heavily criticized by Elizabeth Stanton. really the descent of a landscapist like Claude Monet. In all
H. failed to inspire many other women to fight for matters concerning technique, they form the direct chain of
that right. Impressionism. As regards design, subject, realism, the study
J. were ruthlessly mocked in all news media outlets. 45 of modern life, the conception of beauty and the portrait, the
Impressionist movement is based upon the old French masters,
PASSAGE III
principally upon Chardin, Watteau, Latour, Largillière, Fragonard,
HUMANITIES: The following passage is adapted from a history Debucourt, Saint-Aubin, Moreau, and Eisen. It has resolutely
of French Impressionist painters originally published in 1903. held aloof from mythology, academic allegory, historical
50 painting, and from the neo-Greek elements of Classicism as
Now I must speak at some length of a painter who, together well as from the German and Spanish elements of Romanticism.
with the luminous and sparkling landscapist Félix Ziem, was This reactionary movement is therefore entirely French, and
the most direct initiator of Impressionist technique. Monticelli surely if it deserves reproach, the one least deserved is that
is one of those singular men of genius who are not connected leveled upon it by the official painters: disobedience to the
5 with any school, and whose work is an inexhaustible source of 55 national spirit. Impressionism is an art which does not give
applications. He lived at Marseilles, where he was born, made
a short appearance at the Salons, and then returned to his native
town, where he died poor, ignored, paralyzed and mad. In order
to live he sold his small pictures at the cafés, where they fetched
CONTINUE
  857
Peterson's ACT® Prep Guide 2018
Practice Test 3

much scope to intellectuality, an art whose followers admit 23. The image in lines 35-40 functions figuratively to
scarcely anything but immediate vision, rejecting philosophy suggest that:
and symbols and occupying themselves only with the consid- A. music had a direct influence on Monticelli’s
eration of light, picturesqueness, keen and clever observation, paintings.
60 and antipathy to abstraction, as the innate qualities of French B. Monticelli never used more than one color in his
art. We shall see later on, when considering separately its prin- paintings.
cipal masters, that each of them has based his art upon some C. Monticelli was a great musical composer as well as
masters of pure French blood. a great artist.
D. the basic elements in Monticelli’s paintings have
Impressionism has, then, hitherto been very badly judged.
great emotional impact.
65 It is contained in two chief points: search after a new technique,
and expression of modern reality. Its birth has not been a spon- 24. According to the third paragraph, Impressionistic art
taneous phenomenon. Manet, who, by his spirit and by the involves:
chance of his friendships, grouped around him the principal I. abstract imagery.
members, commenced by being classed in the ranks of the II. the consideration of light.
70 Realists of the second Romanticism by the side of Courbet; and III. picturesqueness.
during the whole first period of his work he only endeavored F. I only
to describe contemporary scenes, at a time when the laws of G. III only
the new technique were already dawning upon Claude Monet. H. II and III only
Gradually the grouping of the Impressionists took place. Claude J. I, II, and III
75 Monet is really the first initiator: in a parallel line with his ideas
and his works Manet passed into the second period of his artistic 25. It can be inferred from the second paragraph that:
life, and with him Renoir, Degas and Pissarro. But Manet had A. Impressionistic art can provoke strong emotions.
already during his first period been the topic of far-echoing B. no artist can be compared to Monticelli.
polemics, caused by his realism and by the marked influence C. landscape art was too mundane for the
80 of the Spaniards and of Hals upon his style; his temperament, Impressionists.
too, was that of the head of a school; and for these reasons D. Monticelli was not appreciated during his lifetime.
legend has attached to his name the title of head of the
26. The passage suggests that Manet was considered a
Impressionist school, but this legend is incorrect.
Realist because:

21. As it is used in line 21, the word inspired most likely F. of his peculiar temperament.
means: G. he tended to paint contemporary scenes.
H. he was friends with many Realists.
A. animated.
J. he was often confused with Monet.
B. encouraged.
C. exceptional. 27. The main idea of the passage is that:
D. educated.
A. Monticelli is one of the key artists of
22. The author of the passage would most likely agree with Impressionism, though his work was not appre-
which of the following statements? ciated until after his death.
B. Impressionism is an exciting art form that took
F. The use of symbols is not necessarily an integral
some time to develop and it has often been
element of good art.
unfairly judged.
G. Romantic and Classical art is worthless.
C. in terms of subject, realism, modernity, beauty,
H. The finest art depicts imagery from mythology.
and portraiture, Impressionism is based on the old
J. The Impressionists were the most technically
French masters.
advanced artists.
D. Manet was often considered the head of the
Impressionist school, but this is inaccurate.

  858
www.petersons.com
Practice Test 3

28. According to Paragraph 2, Monticelli: refinement, and numerous other circumstances, which, without
I. used a thick brush. historical associations, would be uninteresting or imperfectly
20 understood. As the present condition of nations is the result of
II. often painted jewels.
III. was influenced by music. many antecedent changes, some extremely remote, and others
F. I only recent, some gradual, others sudden and violent; so the state,
G. I and II only of the natural world is the result of a long succession of events;
H. II and III only and if we would enlarge our experience of the present economy
J. I, II, and III 25 of nature, we must investigate the effects of her operations in
former epochs.
29. Based on the information in the passage, which of the
following is a fact rather than an opinion? We often discover with surprise, on looking back into the
A. Monticelli’s use of color was inspired. chronicles of nations, how the fortune of some battle has influ-
B. Monticelli’s style was entirely personal. enced the fate of millions of our contemporaries, when it has
C. All of Monticelli’s pictures provoke astonishment. 30 long been forgotten by the mass of the population. With this
D. Monticelli painted landscapes and “fêtes galantes.” remote event we may find inseparably connected the geo-
graphical boundaries of a great state, the language now spoken
30. The phrase “an art whose followers admit scarcely by the inhabitants, their peculiar manners, laws, and religious
anything but immediate vision” in Paragraph 3 implies opinions. But far more astonishing and unexpected are the
that: connections brought to light, when we carry back our researches
35
F. the Impressionists refuse to discuss their work. into the history of nature. The form of a coast, the configuration
G. spontaneity is the main quality of Impressionist of the interior of a country, the existence and extent of lakes,
art. valleys, and mountains, can often be traced to the former prev-
H. the Impressionists are not honest about their work alence of earthquakes and volcanoes in regions which have
processes. 40 long been undisturbed. To these remote convulsions the present
J. the Impressionists are only influenced by artists of fertility of some districts, the sterile character of others, the
pure French blood. elevation of land above the sea, the climate, and various pecu-
liarities, may be distinctly referred. On the other hand, many
PASSAGE IV
distinguishing features of the surface may often be ascribed to
NATURAL SCIENCE: The following passage is excerpted from 45 the operation, at a remote era, of slow and tranquil causes—to
a book about the principles of geology. the gradual deposition of sediment in a lake or in the ocean, or
to the prolific increase of testacea and corals...
Geology is the science which investigates the successive
changes that have taken place in the organic and inorganic Geology is intimately related to almost all the physical sci-
kingdoms of nature; it inquires into the causes of these changes, ences, as history is to the moral. An historian should, if possible,
and the influence which they have exerted in modifying the 50 be at once profoundly acquainted with ethics, politics, jurispru-
5 surface and external structure of our planet. dence, the military art, theology; in a word, with all branches
of knowledge by which any insight into human affairs, or into
By these researches into the state of the earth and its inhab- the moral and intellectual nature of man, can be obtained. It
itants at former periods, we acquire a more perfect knowledge would be no less desirable that a geologist should be well versed
of its present condition, and more comprehensive views con- 55 in chemistry, natural philosophy, mineralogy, zoology, compar-
cerning the laws now governing its animate and inanimate ative anatomy, botany; in short, in every science relating to
10 productions. When we study history, we obtain a more profound organic and inorganic nature. With these accomplishments, the
insight into human nature, by instituting a comparison between historian and geologist would rarely fail to draw correct and
the present and former states of society. We trace the long series philosophical conclusions from the various monuments trans-
of events which have gradually led to the actual posture of 60 mitted to them of former occurrences. They would know to
affairs; and by connecting effects with their causes, we are what combination of causes analogous effects were referable,
15 enabled to classify and retain in the memory a multitude of and they would often be enabled to supply, by inference, infor-
complicated relations—the various peculiarities of national mation concerning many events unrecorded in the defective
character—the different degrees of moral and intellectual archives of former ages. But as such extensive acquisitions are

CONTINUE
  859
Peterson's ACT® Prep Guide 2018
Practice Test 3

65 scarcely within the reach of any individual, it is necessary that declared that geology was in nowise concerned “with questions
men who have devoted their lives to different departments as to the origin of things.”
should unite their efforts; and as the historian receives assistance
from the antiquary, and from those who have cultivated different 31. The passage mentions that features of Earth may be
branches of moral and political science, so the geologist should the results of:
70 avail himself of the aid of many naturalists, and particularly of A. undisturbed areas of land.
those who have studied the fossil remains of lost species of B. land elevation.
animals and plants. C. ancient cataclysms.
D. natural erosion.
The analogy, however, of the monuments consulted in
geology, and those available in history, extends no farther than 32. According to the passage, the geologist should also
75 to one class of historical monuments—those which may be have a background in:
said to be undesignedly commemorative of former events. The I. botany.
canoes, for example, and stone hatchets found in our peat bogs, II. anatomy.
afford an insight into the rude arts and manners of the earliest III. chemistry.
inhabitants of our island; the buried coin fixes the date of the F. I only
80 reign of some Roman emperor; the ancient encampment indi- G. I and II only
cates the districts once occupied by invading armies, and the H. II and III only
former method of constructing military defenses; the Egyptian J. I, II, and III
mummies throw light on the art of embalming, the rites of
33. The passage suggests that geology was considered a
sepulture, or the average stature of the human race in ancient
branch of mineralogy as recently as:
85 Egypt. This class of memorials yields to no other in authenticity,
but it constitutes a small part only of the resources on which A. the late 1600s.
the historian relies, whereas in geology it forms the only kind B. the early 1700s.
of evidence which is at our command. For this reason we must C. the late 1700s.
not expect to obtain a full and connected account of any series D. the late 1800s.
90 of events beyond the reach of history. But the testimony of
34. It can reasonably be inferred from the passage that
geological monuments, if frequently imperfect, possesses at
geology is all of the following EXCEPT:
least the advantage of being free from all intentional misrep-
resentation. We may be deceived in the inferences which we F. a study of how Earth originated.
draw, in the same manner as we often mistake the nature and G. an area of study similar to the study of history.
95 import of phenomena observed in the daily course of nature; H. an area of study dependent on all the physical
but our liability to err is confined to the interpretation, and, if sciences.
this be correct, our information is certain. J. a means to explain Earth’s present condition.

35. As it is used in line 12, the word trace most likely


It was long before the distinct nature and legitimate objects means:
of geology were fully recognized, and it was at first confounded
A. outline the shape.
100 with many other branches of inquiry, just as the limits of history,
B. make a sketch.
poetry, and mythology were ill-defined in the infancy of civili-
C. a very small amount.
zation. Even in Werner’s time, or at the close of the eighteenth
D. locate and follow.
century, geology appears to have been regarded as little other
than a subordinate department of mineralogy; and Desmarest 36. The reason that geologists depend on naturalists is
105 included it under the head of Physical Geography. But the most due to the:
common and serious source of confusion arose from the notion,
F. difficulty of excavating the rocks, minerals, and
that it was the business of geology to discover the mode in
other materials that geologists study.
which the earth originated, or, as some imagined, to study the
G. reluctance of geologists to share their discoveries
effects of those cosmological causes which were employed by
with each other.
110 the Author of Nature to bring this planet out of a nascent and
H. fact that geologists must often study the things
chaotic state into a more perfect and habitable condition. Hutton
that naturalists have discovered.
was the first who endeavored to draw a strong line of demar-
J. importance of understanding different branches
cation between his favorite science and cosmogony, for he
of moral and political science.

  860
www.petersons.com
Practice Test 3

37. According to the passage, an increase in coral may: 39. Based on information from the passage, historians have
all of the following characteristics EXCEPT:
A. alter Earth’s surface features.
B. impede the understanding of a geological feature. A. knowledge of ethics and politics.
C. cause an increase of Testacea. B. working relationships with other academics.
D. increase the tranquility of Earth. C. insight into human affairs.
D. understanding of natural philosophy.
38. The passage states that historians may study mummies
in order to: 40. Which of the following would be the best example of
violent “antecedent changes” that might affect “the
F. learn the dates in which rulers lived.
present condition of nations”?
G. find out the areas that armies occupied.
H. gain knowledge about the human form. F. The eruptions of volcanoes
J. know how ancient physicians treated the ill. G. Nineteenth-century political revolutions
H. Current wars
J. Presidential elections with unfortunate outcomes

STOP
  861
Peterson's ACT® Prep Guide 2018
Practice Test 3

SCIENCE TEST
35 Minutes—40 Questions

DIRECTIONS: There are seven passages in this test. Each passage is followed by several questions. Read each passage,
select the best answer to each related question, and fill in the corresponding circle on the answer sheet. You may look back
at the passages as often as you need.

You are NOT allowed to use a calculator on this test.

PASSAGE I The following procedure was performed to investigate


whether different resistors acted as ohmic devices in a circuit.
As power is supplied to a circuit, current flows through the circuit.
The circuit was constructed as shown in Figure 1.

Practice Test 3 — science


An ammeter is the device used to measure the current, and many
ammeters measure current in milliamperes (mA). The voltage FIGURE 1
responsible for the current can be measured by a voltmeter and
is measured in volts. When a resistor is placed in a circuit, it
dampens the current flowing through a circuit at a given voltage. Ammeter Power Supply
If there is a linear relationship between current and voltage when
a resistor is placed in the circuit, the resistor is considered an
ohmic device. If the temperature of the resistor changes, then it Voltmeter
is not considered an ohmic device. Some resistors are sensitive
to small external temperature changes and will show a change
in resistance as a result of these temperature changes. These Resistor
resistors are called thermistors. The change in resistance exhibited
by a thermistor can be detected by a change in the observed After each resistor was connected to the circuit, the resistor
current at a given voltage. was submerged in water to detect any changes in temperature
as well as its sensitivity to different beginning temperatures.
The power source was turned on and the voltages of the
power source and the resulting current were recorded. The
voltage was changed several times, and the corresponding
current was noted.

CONTINUE
  863
Peterson's ACT® Prep Guide 2018
Practice Test 3

Table 1 summarizes the results when three different resistors 1. When the temperature is 25°C and the voltage is 4.5
were tested at two different temperatures. In all cases, no change volts, what is the current of the circuit when Resistor B is
in water temperature was observed. used?
A. 81 mA
Table 1 B. 90 mA
C. 420 mA
Voltage Current Current
D. 450 mA
(V) (mA) (mA)
Resistor A 2. If 3.5 volts were used with Resistor A at 27°C,
23°C 25°C approximately what would the current have read?

0.25 25 25 F. 63 mA
G. 70 mA
0.50 50 50
H. 350 mA
1.00 100 100
J. 375 mA
2.00 200 200
3. According to the data in Table 1, which of the following
3.00 300 300
resistors respond linearly to a change in voltage?
4.00 400 400
I. Resistor A
4.50 450 450
II. Resistor B
5.00 500 500
III. Resistor C
Resistor B
A. I only
23°C 25°C
B. II only
0.25 150 150
C. II and III
0.50 195 195 D. I and III
1.00 230 230
4. Resistor B does NOT appear to be a thermistor because:
2.00 295 295
F. at low voltages, Resistor B produces higher currents
3.00 345 345
than either of the other resistors.
4.00 405 405
G. at higher temperatures, Resistor B produces lower
4.50 420 420 currents than either of the other resistors.
5.00 445 445 H. there is a change in resistance at different voltages.
Resistor C J. there is no change in resistance at different
temperatures.
23°C 25°C
0.25 5 4.5 5. What additional information would be necessary to
conclude whether Resistor C is a thermistor?
0.50 10 9.0
1.00 20 18.0 A. The current-voltage response of Resistor C at a third
temperature
2.00 40 36.0
B. The temperature of Resistor C at various voltages
3.00 60 54.0
C. Whether the current of Resistor C responds linearly
4.00 80 72.0 with temperature
4.50 90 81.0 D. There is enough information to determine whether
5.00 100 90.0 Resistor C is a thermistor.

  864
www.petersons.com864
Practice Test 3

6. Which of the following hypotheses would be disproved


if there had been a noted temperature change in the
water for all six trials?
F. Resistor A is an ohmic device because of its linear
relationship between voltage and current.
G. Resistor B is affected by different starting
temperatures.
H. Resistor C is not an ohmic device because it does
not have a linear relationship between voltage
and current.
J. Resistor C is not affected by different starting
temperatures.

PASSAGE II

Seagrasses are plants that grow along the coasts of warm, shallow seas. They form underwater meadows that improve water quality,
help prevent erosion of sand and sediment during storms, and provide shelter and food for fish and other aquatic animals. Ecologists
have discovered that they also store a large amount of carbon and other nutrients in the sediment. The ecologists were interested
in how much.

Ecologists selected an area that was undergoing restoration after the original seagrass meadow was wiped out during a severe
hurricane. Seagrass had been seeded onto two separate meadows; one meadow had been reseeded four years ago, and the other
had been reseeded ten years ago. From each meadow, the ecologists took four 20-cm deep, 10-cm diameter sediment cores. They
also took four similar cores from the unplanted sediment immediately around each of the meadows. They removed shells, rocks, and
large roots from the sediment, weighed them wet and again after drying to determine bulk density, and determined the percentages
of organic matter, carbon, and nitrogen in the sediment. The averages of their results are summarized in Table 1.

Table 1
Seagrass
% Bulk
Meadow Density % %
Site Organic Density
Age (years) (shoots Carbon Nitrogen
Matter (g/cm3)
/m2)
Site 1 0 0 0.4 0.2 1.6 1.53
Site 1 4 123 0.39 0.2 1.6 1.44
Site 2 0 0 0.36 0.86 1.4 1.61
Site 2 10 429 0.52 3.75 1.9 1.30

CONTINUE
  865
Peterson's ACT® Prep Guide 2018
Practice Test 3

After examining nutrient accumulation, the ecologists wanted 9. Which of the following best explains why the scientists
to determine which physical characteristics of the seagrass collected sediment from an unplanted area at each site?
meadow are most helpful in stabilizing sediment to prevent A. To control for differences in original sediment
erosion. They placed four sediment traps within the 10-year composition between the two sites
old meadow, and recorded the characteristics of the seagrass B. To control for differences in water depth between
around the sediment trap: the shoot density (in shoots/m2 of the two sites
seabed), leaf mass (grams of seagrass leaves/ m2 of seabed), C. To determine the most effective seagrass density
seagrass height (cm), and leaf surface area (m2 leaf surface/m2 for nutrient sequestration
of seabed). After each day, they weighed the amount of D. To determine the rate of nutrient loss in unvege-
sediment collected in each sediment trap (g/m2 of trap area, tated seabeds
per day). They compared the average of six days of sediment
results with each of the seagrass characteristics in Table 2. 10. Which hypothesis is supported by the data in Table 2?
F. Taller seagrass plants reduce turbulence in a larger
Table 2 portion of the water column, resulting in greater
sediment deposition.
Trapped Seagrass G. More densely planted seagrass meadows prevent
Seagrass Seagrass
Sediment Surface Seagrass sand from settling to the seabed, resulting in
Density Mass
Weight Area Height reduced sediment deposition.
(shoots/m2) (g/m2)
(g/m2/day) (m2/m2) H. Greater seagrass mass increases turbulence in the
3.3 200 6.0 1.8 29 water column, resulting in reduced sediment
3.8 170 10.2 2.8 56 deposition.
4.1 260 6.8 3.0 33 J. Greater seagrass surface area more effectively
dissipates the energy of ocean currents, resulting in
4.8 230 10.0 4.1 52
greater sediment deposition.

11. A local company wants to dredge a nearby seagrass


7. According to the data in Table 1, the greatest difference meadow to collect sand for a public beach. To decrease
in sediment bulk density was found between: the negative impacts of the dredging, they would like to
replant seagrass afterwards. What is the most likely
A. years 0 and 4 at Site 1. outcome of replanting?
B. years 0 and 10 at Site 2.
C. the two sites without seagrass. A. After 10 years, the replanted seagrass meadow
D. the two sites with seagrass. would have a higher shoot density than the original
one.
8. According to the data, the biggest relative change B. The percentage of organic matter found in the
between 0 and 10 years was found in: sediment would be unchanged by dredging and
F. % carbon. replanting.
G. % nitrogen. C. There would be immediate storage of carbon and
H. % organic matter. nitrogen in the sediment, similar to that in the
J. bulk density. original meadow.
D. There would be a lag of more than four years before
carbon and nitrogen storage reached that of the
original meadow.

  866
www.petersons.com866
Practice Test 3

12. Scientists have measured atmospheric carbon dioxide


(CO2) levels that are higher now than they have been in
any measured year and estimated to be higher than any
time in the last 400,000 years. If ecologists want to
increase the amount by which seagrass meadows take
up CO2 and store it in the sediment, what action is most
likely to be effective?
F. Protect and expand existing seagrass meadows.
G. Allow increased fertilizer runoff into coastal
waterways.
H. Harvest and replant seagrass meadows every 10
years.
J. Plant native plants along the shore near seagrass
meadows.

PASSAGE III

Elements are organized into a periodic table to help understand their similarities and differences. They are arranged by the number
of protons (positively charged particles) in the nucleus, which increases along each row of the periodic table. In its neutral state, each
element has an equal number of protons and electrons—negatively charged particles that orbit the nucleus in different energy
levels called orbitals. Elements in each column of the periodic table share the same number of electrons in the highest (outermost)
energy level. Elements in the rightmost column, the noble gases, have a completely filled outermost energy level and are relatively
stable and unreactive elements. They are gases at room temperature and pressure and have slightly different characteristics.

Table 1 contains data for the noble gases, including atomic number, atomic weight (in atomic mass units, amu) atomic radius (in
picometers, pm), density, and boiling point.

Table 1
Atomic Atomic Boiling
Atomic Density
Element Weight Radius Point
Number (g/L)
(amu) (pm) (°C)
Helium 2 4.0 31 0.18 –269
Neon 10 20.2 38 0.9 –246
Argon 18 39.9 71 0.86 –186
Krypton 36 83.8 88 3.75 –153
Xenon 54 131.3 108 5.9 –108
Radon 86 222 120 9.73 –62

13. The element with atomic radius of 88 pm has a boiling 14. According to the table, which element has an atomic
point of: weight approximately four times that of neon?
A. –62°C. F. Helium
B. –108°C. G. Krypton
C. –153°C. H. Xenon
D. –246°C. J. Radon

CONTINUE
  867
Peterson's ACT® Prep Guide 2018
Practice Test 3

15. Data in the table indicates that as the atomic number PASSAGE IV
increases, the boiling point:
Sedimentary rocks are formed when sediment—sand, dirt, min-
A. increases. erals, and organic particles—settle into place and are gradually
B. decreases. compressed and cemented together. The relatively mild condi-
C. remains constant. tions of this rock-forming process make it ideal for the preser-
D. does not vary systematically. vation of fossils. Geologists can use sedimentary rocks to
16. One hypothesis is that as atomic weight increases, determine an area’s geologic history, including the different
density will increase as well. Which pair of elements environments that an area experienced over time. Figure 1 shows
supports this hypothesis? different layers of sedimentary rock exposed at the edge of a
canyon. The composition of rock, approximate age, thickness,
I. Neon and argon and depth of each layer are listed in Table 1.
II. Krypton and xenon
III. Helium and radon 0
1
F. I and II 2
3
G. II and III
H. II only
J. III only 4
500

Depth below surface (m)


17. A new element, oganesson, with atomic number 118
has been discovered to lie in the same periodic column
as the noble gases. Based on the information in this
passage, what would be the best prediction for 5
oganesson’s atomic radius and boiling point? 1000 6
7
A. Greater than 120 pm and greater than –62°C
8
B. Less than 120 pm and less than –62°C
C. Greater than 120 pm and less than –62°C
D. Less than 120 pm and greater than –62°C
1500 9

10

18. What is the relationship between thickness and depth of


the rock layers?
F. Thicker layers are typically found at greater depths.
G. Thicker layers are typically found at shallower
depths.
H. Rock layers less than 50 m thick are only found at
depths of less than 500 m.
J. There is no relationship between thickness and
depth of rock layers.

  868
www.petersons.com868
Practice Test 3

Table 1

Rock/Layer Age Thickness Depth


Layer
Composition (Mill yrs) (m) (m)

Conglomerate,
1 100 30 10.0
Sandstone
Limestone, Mudstone,
2 130 80 0–30
Sandstone, Gypsum
3 Sandstone 160 30 30–110
4 Sandstone 170 670 110–140
Siltstone, Sandstone,
5 180 150 140–810
Mudstone
Sandstone, Freshwater
6 190 40 810–960
Fish Fossils
Siltstone, Sandstone,
7 200 110 960–1,000
Mudstone, Shale
Shale, Conglomerate,
Gypsum, Limestone,
8 220 160 1,110–1,270
Sandstone, Quartz,
Petrified Wood
Shale, Sandstone,
9 Limestone, Siltstone, 230 550 1,270–1,820
Mudstone, Gypsum
10 Limestone, Chert 260 * 1,820

* There is no thickness listed for Layer 10 because the thickness cannot be measured
without excavating.

19. Limestone is composed mainly of compressed skeletal 21. Carnotite is a bright greenish-yellow mineral found
fragments from sea organisms such as coral, forams, and around petrified wood. It is a uranium ore and is
mollusks. It is high in calcium carbonate-containing radioactive. Assuming the sedimentary rock layers are
minerals like calcite and aragonite. During which of the uniform as they extend away from the canyon, approxi-
following time periods is it most likely that the region of mately how deep would miners have to excavate in the
the canyon was covered by a shallow sea? surrounding area in order to obtain uranium?
A. 60 million years ago A. 160 m
B. 100 million years ago B. 500 m
C. 130 million years ago C. 1,200 m
D. 200 million years ago D. 2,000 m

20. Which measurement is shown on the vertical axis of 22. Which conclusion is NOT supported in the data?
Figure 1?
F. The age of the rock layers increases with depth.
F. Temperature G. The most common rock type in the canyon is
G. Age of rock sandstone.
H. Depth below the canyon surface H. Thicker rock layers correspond to longer time
J. Thickness of rock layers periods.
J. Gypsum is only found in layers that also contain
limestone.

CONTINUE
  869
Peterson's ACT® Prep Guide 2018
Practice Test 3

23. Geologists have found small remnants of a rock called upon consuming. Lightweight muscular body structure would
welded tuff exposed at the surface (above layer 1) near be crucial to the success of this type of predator.
the canyon. Welded tuff forms when rock and dust is
Based upon this observation, along with a number of obvious
exposed to temperatures over 1,000°C, hot enough that
physical similarities and evidence from the fossil record, we are
the rock welds together. What does this suggest about
convinced that birds evolved from small, carnivorous dinosaurs
the area near the canyon?
called theropods. A mere examination of the forelimb, hindlimb,
A. The area experienced a hot, tropical climate around and feet of a theropod fossil, and a comparison to one of the five
100 million years ago. available specimens of Archaeopteryx*, will bear this out. In
B. A volcano erupted in the area less than 100 million addition, more recent discoveries of fossil dinosaurs with bird-like
years ago. traits and habits, particularly the finds uncovered in the Liaoning
C. The area was covered by sea at least twice in the province of China, lend further credible support for our position
last 260 million years. that birds are for all intents and purposes actual members of the
D. A volcano erupted in the area more than 260 lineage Dinosauria living and thriving in our midst.
million years ago and then went extinct.
*Archaeopteryx was a feathered reptile of the late Jurassic Era thought to
PASSAGE V represent an intermediate form between dinosaurs and birds.

The apparent bird-dinosaur evolutionary connection has been 24. According to Paleontologist A, similarities in the body
a source of considerable debate among paleontologists during forms of dinosaurs and birds:
the second half of the twentieth century. This association was
F. represent a failed experiment of evolution.
proposed on the basis of numerous anatomical similarities and
G. are the products of convergent evolution.
has been supported by the discovery of fossils of a small number
H. are completely without significance.
of seeming transitional forms uncovered in Europe and Asia. Yet
J. helped both types of organism survive a large mass
scientists differ in their interpretation of the significance of these
extinction.
similarities and the nature of the fossil evidence as well.
25. Which of the following types of evidence, if found,
Paleontologist A
would lend strong support to the position of
The discovery of fossil reptiles equipped with feathers, wings, Paleontologist A?
and beak-like snouts may be significant but more likely provides A. Discovery of thecodont fossils with characteristics
only limited support for the dinosaurs-into-birds hypothesis. of modern birds and existing dinosaur fossils
Convergent evolution often provides animals of very distant B. Discovery of another possible intermediate form
lineages with similar appendages—witness, for example, the between dinosaurs and birds from the Jurassic Era
similarities in the body shape and presence of fins in fish and C. Discovery of an avian prototype dating back to
cetaceans such as whales and dolphins. We would never put before the beginning of the era of dinosaur
forth the idea that orcas evolved from sharks based on the mor- dominance
phological similarities of these creatures; it would be immediately D. A careful examination of several sets of theropod
deemed absurd. fossil remains
It is more likely the case that birds and dinosaurs share a very 26. Which of the following is a criticism that Paleontologist
distant common ancestor, perhaps from among the thecodonts. A would make of the avian evolutionary hypothesis of
These prototypical reptiles of the late Permian survived the largest Paleontologist B?
mass extinction recorded in the planet’s history to bring forth
many more recent lines; crocodiles, dinosaurs, pterosaurs, and F. It ignores the possibility of the existence of
birds are the most notable among these. transitional forms.
G. It ignores the impact of a very large mass
Paleontologist B extinction.
In our studies of numerous dinosaur fossils, it has become obvious H. It assumes that morphological similarities are a
that the lifestyles of dinosaurs were amazingly varied. No longer result of a direct lineage.
is it acceptable to view dinosaurs only as lumbering, cold-blooded J. It proposes that dinosaurs and birds arose from
monsters; indeed, the most frightening dinosaurs did not lumber distant lineages.
at all. They were agile, swift, and deadly predators who could
run, leap, kick, and shred to pieces an animal they were intent

  870
www.petersons.com870
Practice Test 3

27. Which of the following perspectives would be consistent PASSAGE VI


with the views of Paleontologist B?
Ecology graduate students wished to experiment with levels of
A. Convergent evolution produces similar forms in diversity in a simple community to observe the relationship
diverse lineages. between increasing complexity and stability of populations. In
B. Dinosaurs and birds may be related via a common particular, they were interested in the impact of changing certain
ancestor. conditions in a community on populations of two species of
C. Birds and dinosaurs arose from completely separate Paramecium, a common ciliated protozoan.
lineages.
A trophic level is the number of steps a species is away from the
D. Birds arose from a lineage of dinosaurs.
producer species in a community. Producers are organisms that
28. If genetic evidence were established to date the avian synthesize energy out of chemical products into nutrients. Table
lineage 85 million years prior to the rise of 1 shows the trophic level occupied by each type of organism
Archaeopteryx, this finding would tend to: involved in the experiments and the number of species that
F. support the theory of Paleontologist A. would be used on each level.
G. support the theory of Paleontologist B.
H. support the theories of both paleontologists. Table 1
J. refute the theories of both paleontologists.
Number of
Trophic Level Organism
29. If Paleontologist B could confirm that birds appeared Species Studied
much later in evolutionary history than any dinosaurs,
First Bacteria 3
which of the following statements would reconcile this
fact with the theory of Paleontologist A? Second Paramecia 2

A. The ancestors of birds and the ancestors of Amoebae


dinosaurs were exposed to specific environmental Third (predator of 2
conditions at the same time, and this caused the protozoa)
development of similar characteristics.
B. The ancestors of birds and the ancestors of
dinosaurs were exposed to specific environmental Experiment 1
conditions that caused the development of similar
The graduate students wished to study the relationship between
characteristics, but the dinosaur ancestors were
one species of Paramecium and the number of species of bacteria
exposed to these environmental conditions later
available for consumption in the community. They created 300
than the bird ancestors were.
“microcosms”—100 cultures each populated with communities
C. The rate of evolutionary change from the the-
of one, two, or three species of bacteria and one of the Paramecium
codont ancestor was much slower for the lineage
species. After 20 days, they examined the cultures individually.
that resulted in birds than for the lineage that
The results are displayed in Table 2.
resulted in dinosaurs.
D. The rate of evolutionary change from the the-
codont ancestor was much faster for the lineage Table 2
that resulted in birds than for the lineage that Number of Cultures in
resulted in dinosaurs. Number of Bacterium
Which Paramecia
Species
Survived
1 32/100

2 61/100

3 70/100

CONTINUE
  871
Peterson's ACT® Prep Guide 2018
Practice Test 3

Experiment 2 30. What was the dependent variable in Experiment 2?

The students then decided to study how two species of F. The number of species of bacteria
Paramecium would be affected when different combinations of G. The number of species of Paramecium
Paramecium and bacteria were mixed in the cultures. They created H. The length of time after which cultures were
100 dishes each of six separate types of communities—600 examined
cultures with different combinations of the Paramecium and the J. The number of cultures with surviving Paramecium
three bacteria species. After 20 days, they examined the cultures 31. According to the results of Experiments 1 and 2,
and recorded their results. These results are reproduced in Table 3. increasing the number of bacteria species present in
the community:
Table 3 A. decreased the survival rates of the paramecia in
Number of Cultures in the community.
Number of Bacterium/
Which Paramecia B. increased the survival rates of the paramecia in the
Paramecium Species
Survived community.
1/1 35/100 C. had no effect on any species in the community.
D. decreased the survival rates of one type of bacteria
2/1 58/100
in the community.
3/1 65/100
1/2 20/100 32. According to the results of Experiment 2, increasing the
2/2 26/100 number of Paramecium species in the community was
related to:
3/2 31/100
F. decreased survival rates for the paramecia,
depending on the number of bacteria species
Experiment 3 present in the community.
G. decreased survival rates for all species present in
The last condition that the graduate students studied was the
the community.
addition of a third trophic level to their microcosms. They intro-
H. decreased survival rates for the paramecia,
duced two different species of Amoebae that feed on Paramecium.
independent of the number of bacteria species
They decided to use five different versions in 100 culture dishes
present in the community.
each, creating 500 communities for this last experiment. They
J. increased survival rates for the Paramecium,
allowed them to grow undisturbed for 20 days, and then examined
depending on the number of bacteria species
the cultures and recorded their results in Table 4 displayed below.
present in the community.

Table 4 33. What new factor was introduced in Experiment 3?


A. An additional species of bacteria
Number of Bacterium/ Number of Cultures in
B. A longer time for incubation of the experimental
Paramecium/ Which Paramecia
cultures
Amoeba Species Survived
C. A new method for culturing the experimental
1/1/1 22/100
microbes
2/1/1 15/100 D. An additional trophic level
2/2/1 8/100
34. After examining the results of Experiment 3, it would be
2/2/2 6/100 reasonable to conclude that increased diversity in the
3/2/2 2/100 experimental communities:
F. was beneficial to all species present in the
communities.
G. had no effect on any species in the communities.
H. had a detrimental effect on the survival rate of one
species of Amoeba only.
J. had a detrimental effect on the survival rates of
both species of Paramecium under study.

  872
www.petersons.com872
Practice Test 3

PASSAGE VII

Classification of planets in our solar system has been a controversial issue, especially in the last decade. Advanced telescopes have
allowed scientists to see more celestial objects in space, so scientists have observed characteristics among the objects that have
caused them to reconsider whether certain planets should retain their “planet” status. Some of the characteristics that make these
objects distinct include size, mass, and shape, as well as orbital characteristics. Two orbital characteristics of planets often compared
by astronomers are orbital inclination and orbital eccentricity. Orbital inclination is the amount an object’s orbit is tilted with respect
to the plane of the solar system. Orbital eccentricity refers to the amount that an object’s orbit deviates from a circular orbit.

In 2006, the International Astronomical Union (IAU) officially defined what a planet is and established a system of classification for
celestial bodies other than satellites (moons). According to the definition, a planet is a celestial body that orbits the sun, has sufficient
mass to be rounded in shape by its own gravity, and has cleared out other large rocks and bodies from its orbital path. The other
classifications established were dwarf planets and small solar system bodies. Based on these new definitions, Pluto was reclassified
from a planet to a dwarf planet.

The table below summarizes many of the qualities of the eight planets in our solar system.

Table 1

Inner Planets Outer Planets


Celestial
Mercury Venus Earth Mars Jupiter Saturn Uranus Neptune
Body
Mass
(kg) 3.3 × 1023 4.9 × 1024 6.0 × 1024 6.4 × 1023 1.9 × 1027 5.7 × 1026 8.7 × 1025 1.0 × 1026

Mean
Density 5,427 5,204 5,520 3,933 1,326 687 1,318 1,638
(kg/m3)
Black-body
Temp. 442.5 238.9 247.3 216.6 90.6 63.9 35.9 33.2
(°K)
Orbital
Inclination 7.000 3.390 0.000 1.850 1.305 2.484 0.770 1.769
(°)
Orbital
Eccentricity 0.2056 0.0068 0.0167 0.0934 0.0484 0.0542 0.0472 0.0086
(°)
Rotation
Period 1,407.60 5,832.50 23.93 24.62 9.93 10.50 17.24 16.11
(hrs.)

35. Uranus has a mass greater than which of the following 36. Black-body temperature is the temperature that would
planets? result from a planet absorbing all received electromag-
netic radiation without reflection. If black-body
A. Mars, Jupiter, and Saturn
temperatures are higher for objects closer to the sun,
B. Neptune, Mercury, and Earth
which of the following would be considered farthest
C. Venus, Mars, and Saturn
from the sun based on the data presented?
D. Mercury, Earth, and Mars
F. Mercury
G. Uranus
H. Jupiter
J. Neptune
CONTINUE
  873
Peterson's ACT® Prep Guide 2018
Practice Test 3

37. If an astronomer proposed that a celestial body does not 39. It was hypothesized that Jupiter, Saturn, Neptune, and
qualify as a planet based solely on having an orbital Uranus were gaseous planets, whereas the others were
inclination greater than 5 degrees and orbital eccen- composed of solid material. Which of the following
tricity greater than 0.2, how many of the current planets statements best supports this hypothesis?
would lose their planet classification? A. Jupiter, Saturn, Neptune, and Uranus have lower
A. four temperatures than most of the other planets.
B. three B. Jupiter, Saturn, Neptune, and Uranus have densities
C. two much smaller than the densities of the
D. one other planets.
C. Jupiter, Saturn, Neptune, and Uranus have greater
38. The orbital eccentricity of a celestial body refers to the
masses than the other planets.
amount that its orbit deviates from a perfectly circular
D. Jupiter, Saturn, Neptune, and Uranus have rotation
orbit. A value of 0 is a circular orbit. The higher the
periods that are much shorter than the
eccentricity value, the more distorted the orbit, which
other planets.
can result in objects displaying elliptical, parabolic, and
hyperbolic orbits. 40. Which of the following statements is supported by the
data?
Which pair lists the planet with an orbit that deviates
the least from a perfect circle and the one that F. Rotational period increases as mass increases.
deviates the most? G. The two planets with the highest mass also have
the lowest densities.
F. Earth; Mercury
H. The two planets closest in mass are also closest in
G. Mercury; Neptune
rotational period.
H. Venus; Mercury
J. The planet with the lowest mass has the highest
J. Neptune; Venus
orbital eccentricity.

STOP
  874
www.petersons.com874
Practice Test 3

WRITING TEST
40 Minutes

DIRECTIONS: This is a test of your writing skills. You have forty (40) minutes to write an essay in English. Be sure to carefully
read the issue and three perspectives presented before planning and writing your essay so you understand clearly the task
you are being asked to do. Your essay will be graded on the evidence it provides of your ability to analyze the issue; evaluate
and analyze the perspectives; state and develop a personal perspective; and describe the relationship between the given
perspectives and your own, while effectively using organization, logic, and language according to the conventions of standard
written English.

Alternative Energy

Practice Test 3 — writing


The United States—and the entire world—is at a crossroads when it comes to our sources of energy. Traditional fossil fuels, including
coal, petroleum, and natural gas, are our primary fuel sources, and are responsible for everything from powering our vehicles to
warming our homes and providing electricity for our favorite gadgets. However, fossil fuels are also responsible for emitting carbon
dioxide into the atmosphere, which most climate scientists feel adversely impacts the environment and contributes to global
warming. As a result, some have called for increased exploration and investment in renewable, non-fossil sources of energy,
including hydroelectric, solar, nuclear, and wind power, which are better for the environment and would also help reduce our
dependence on foreign oil supplies. Efforts to find new, more efficient, and more cost-effective methods for harnessing the power
of these alternative energy sources is already underway. As we continue to march through the twenty-first century, what are your
thoughts on the advantages and disadvantages of alternative energy exploration and use? What roles should we play regarding
strategies, investment, and incentives for alternative energy use?

Read and carefully consider these perspectives. Each suggests a particular way of thinking about if and how alternative energies should
be used to meet the needs of the country

Perspective One

The United States has sufficient fuel from coal, oil, and gas, and we should continue to use these fossil fuels for all of our
energy needs. The media exaggerate their negative effects and the science isn’t definitive. Our nation has thrived through
the twentieth century thanks to these abundant and effective sources of power, and the government shouldn’t waste
time, energy, and resources on alternative energies, which are expensive, inefficient, and unable to fully meet the needs
of the country.

Perspective Two

A serious and committed investment in alternative energy is not only needed right now, it’s long overdue. Our reliance
on fossil fuels has kept us dependent on energy sources that are destroying our planet. We cannot afford any further
apathy, or to put off figuring out how to free ourselves from fossil fuels—even if it’s expensive and difficult. We need to
end our reliance on fossil fuels immediately. Our planet is worth the effort.

Perspective Three

A decision as large and important as the direction of energy sourcing in the United States should not be made impulsively.
Available scientific evidence indicates that a continued long-term reliance on fossil fuels as our primary energy sources
might not be the most responsible approach. However, quitting fossil fuels “cold turkey” or without a clear plan isn’t
realistic either. Our nation needs to develop a long-term plan that includes a tapering off from fossil fuels and an increased
use of alternative, renewable energy sources. Furthermore, we need to work with other responsible countries around
the world on decreasing fossil fuel reliance; only a worldwide effort will help us tackle this issue.

  875
Peterson's ACT® Prep Guide 2018
Practice Test 3

Essay Task
Write a unified, coherent essay in which you evaluate multiple perspectives on alternative energy. In your essay, be sure to:

•  examine and assess the perspectives given


•  declare and explain your own perspective on the issue
•  discuss the relationship between your perspective and those given
Your perspective may be in full or partial agreement, or in total disagreement, with any of the others. Whatever the case, support
your ideas with logical reasoning and detailed, persuasive examples.

Plan and Write Your Essay


Use the first two lined pages of the Writing test answer sheet (found at the beginning of the test) to generate ideas and plan
your essay. Then use the following four pages to write your essay. Consider the following as you compose your essay:

What are the strengths and weaknesses of the three perspectives provided?

•  Identify the insights they present and what they fail to consider.
•  Ascertain why a given perspective might persuade or fail to persuade.
How can you apply your own experience, knowledge, and values?

•  Express your perspective on the issue, identifying the perspective’s strengths and weaknesses.
•  Formulate a plan to support your perspective in your essay.

  876
www.petersons.com876
Answer Keys and Explanations

English
1.   B 20.   J 39.   D 58.   J
2.   F 21.   D 40.   H 59.   A
3.   B 22.   G 41.   B 60.   H
4.   F 23.   C 42.   J 61.   C
5.   C 24.   G 43.   C 62.   J
6.   J 25.   B 44.   F 63.   D
7.   C 26.   F 45.   C 64.   H
8.   G 27.   C 46.   J 65.   D
9.   A 28.   H 47.   B 66.   F
10.   J 29.   D 48.   H 67.   C
11.   A 30.   F 49.   B 68.   F
12.   F 31.   B 50.   F 69.   B
13.   C 32.   F 51.   B 70.   J
14.   G 33.   B 52.   G 71.   A
15.   B 34.   G 53.   B 72.   H
16.   J 35.   D 54.   F 73.   D
17.   B 36.   J 55.   C 74.   F
18.   H 37.   C 56.   J 75.   C
19.   B 38.   G 57.   A

1. The correct answer is B. As written, the verbs in the 5. The correct answer is C. The conjunction and is
sentence are not in proper parallel form. Both of the needed to correct the comma splice in the original
verbs in the underlined portion of the sentence need to sentence, which is the incorrect use of just a comma
be in the present tense, so choice B is correct. The other to connect two independent clauses. A comma
answer choices have to look and to stare in inappro- before the and is also needed, because this con-
priate tenses for this sentence. junction is being used to coordinate two inde-
pendent clauses with two separate subjects, so
2. The correct answer is F. With is the appropriate
choice B is incorrect. The conjunction yet (choice D)
preposition in this common phrase. Choice G introduces
is inappropriate, because there is no change in
a grammatical error, and choice H changes the meaning
direction between the two clauses.
of the sentence. The phrase should not be omitted
because it explains the writer’s trembling, so choice J 6. The correct answer is J. The past tense is appro-
is incorrect. priate because the writer is describing his childhood.
Choices F, G, and H are all written in the
3. The correct answer is B. The verb here refers to an
present tense.
action (staring at the green shoots) that began in the
immediate past and continues into the present; 7. The correct answer is C. This option clearly states
therefore, the present perfect tense is appropriate. The the writer’s point that in New York, culture was the
original sentence mistakenly uses the past perfect dominating element, even when it came to marking
tense, so choice A is incorrect. Choice C would be used the passage of the seasons. Choices A, B, and D are
only if the sentence were written in the simple present all worded confusingly.
tense, and choice D would be used if it was written in
8. The correct answer is G. In the sentence, the phrase
the simple future tense.
the gradual way one transformed into another
4. The correct answer is F. The verb tense of the under- primarily functions as an example of one of the
lined portion of this sentence is correct as written. The particulars with which the author was not familiar.
other answer choices contain incorrect verb forms for The phrase is merely an example of one particular
this sentence. that eluded the author; it is not proof that he did not
understand the seasons, so choice F is not the best

  877
Peterson's ACT® Prep Guide 2018
Answer Keys and Explanations

answer. The phrase is not particularly poetic and serves wrong; there is specific information about the life cycle
a more important function than merely making the of chrysanthemums in the third paragraph.
essay more appealing to read, so choice H is not the
15. The correct answer is B. This detail is an example of
best answer, either. The phrase does not serve as a
how little the writer knew about nature; therefore, it
transition between two different yet related ideas, so
belongs in the paragraph that discusses growing up in
choice J is incorrect.
New York. Paragraphs 1 (choice A), 3 (choice C), and 4
9. The correct answer is A. Paragraph 2 tells us that the (choice D) all take place in the present.
writer grew up knowing little about nature. This fact is
16. The correct answer is J. The idiomatic expression is: as
essential to understanding the garden’s effect on the
x goes, so goes y, so choice J is correct. Choices F and G
writer. The writer’s current environment is quite
add unnecessary words. Choice H is simply awkward.
different from a city, but he never suggests the city was
something he felt lucky to have escaped, so choice B is 17. The correct answer is B. As these words are most
wrong. Paragraph 2 may be a vivid recollection (choice commonly used, affect is the proper verb form, while
C) or an indication of the writer’s sensitivity (choice D), effect is the noun form. Here the word is a noun, so the
but neither of these describes the paragraph’s main proper word to use is effect, which means you can
purpose. eliminate choices A and D. The preposition that properly
follows effect is on, so choice B is correct and choice C is
10. The correct answer is J. As written, the underlined
incorrect.
portion of the sentence contains an incorrect, com-
monly confused word—too. Its homonym, two, is the 18. The correct answer is G. The writer is speaking about
appropriate word choice here, so choice J is correct. our present understanding of trade in the past, so
Choice G contains the incorrect too and an inappro- choice G is correct, and you can eliminate choices F and
priate possessive form of season. Choice H contains H, which are in the past tense. It is an assertion without
another incorrect homonym, to. any sense of the conditional, so choice J is incorrect.

11. The correct answer is A. No comma should be used 19. The correct answer is B. This is the only sentence that
before yellow because bright modifies yellow and all the stresses the historical importance of exchange and
other colors. No commas are needed to separate the thereby links the previous paragraph to the examples of
colors because the writer has chosen to link them with this phenomenon presented in Paragraph 2. Choice A is
the conjunction and. Therefore, choices C and D are inappropriate, since agriculture is merely one detail in
incorrect. Choice B, however, fails to link yellow and red Paragraph 2 and not its main idea. Choice C would be a
with either a comma or and, so it is incorrect. more fitting concluding sentence for Paragraph 1 than
an introductory sentence for Paragraph 2, since a phrase
12. The correct answer is F. The present tense is appro-
such as “Many examples of this…” should continue a
priate for a general situation that is repeated year after
previous idea directly. Choice D introduces that same
year. The plural, Daffodils, demands the third-person
organizational error and focuses on the modern world,
plural form of the verb. Choices G and J are in the past
while Paragraph 2 focuses on the historical importance
tense. Choice H is a singular verb when a plural one is
of exchange.
required.
20. The correct answer is J. The clearest construction lists
13. The correct answer is C. As written, this sentence
the subjects and proceeds directly to the verb without
creates a sentence fragment. There is a need for internal
punctuation or unnecessary words. As originally written,
sentence punctuation here, so choices B and D are
the list lacks the conjunction and between the last two
incorrect.
items in the list and misuses a comma to separate the
14. The correct answer is G. In the third paragraph, the list from the rest of the sentence. In choice G, a colon is
author uses colorful detail to describe when chrysan- correctly used instead of a comma, but the conjunction
themums bud, when they bloom, and when they die, and is still missing. Choice H changes the meaning of
while relating the timeline to the change of seasons, so the sentence.
choice G is the best answer. Choice F is not the best
21. The correct answer is D. No punctuation should
answer, since it provides no details about planting
separate the noun from the prepositional phrase that
chrysanthemums. Choice H is true in itself, but it does
identifies it. Choices A, B, and C all contain unnecessary
not answer this particular question. Choice J is simply
punctuation.

  878
www.petersons.com
Answer Keys and Explanations

22. The correct answer is G. The present perfect tense 31. The correct answer is B. As written, the sentence is a
indicates the connection between the past and the run-on and needs to be fixed. Choice B correctly
present, between antiquity and today. Choices F, H, and combines the two independent clauses while elimi-
J are all in the wrong tense. nating the run-on sentence. Choice C places the
semicolon in the wrong place in the sentence. Choice D
23. The correct answer is C. Greeks is the subject of the
incorrectly creates a fragment by inserting a period.
relative clause, so the relative pronoun who is appro-
priate. Whom is a personal pronoun, so the original 32. The correct answer is F. Paragraph 1 discusses
sentence is incorrect. Choices B and D are inappropriate everyday innovations that we take for granted;
because they refer to non-human subjects. Paragraph 2 jumps into discussing aspirins. Choice F
would make an effective transition sentence from
24. The correct answer is G. A comma is required before a
Paragraph 1 to Paragraph 2. The other choices are
coordinating conjunction when linking two inde-
off-target and would not serve as effective transitions.
pendent clauses. And is the best choice because it
indicates the non-contrasting relationship between the 33. The correct answer is B. No punctuation is required at
clauses, so choices F and H are incorrect. Therefore also this position in the sentence, so choice B is correct. The
expresses this relationship but cannot be used to link other answer choices create sentence fragments and
two independent clauses with only a comma; a are therefore incorrect.
semicolon would be required instead, so you can
34. The correct answer is G. Since countless and innu-
eliminate choice J.
merable have the same meaning, including both words
25. The correct answer is B. This phrasing properly in this sentence is redundant. Choice G eliminates the
indicates that the question is about the character of redundancy and is therefore the correct answer. Choice
modern Italy. Choices A, C, and D ask different H simply reverses the words and does nothing to
questions. eliminate the redundancy. Choice J substitutes in two
other redundant words and fails to eliminate the
26. The correct answer is F. The imperative form avoids
problem.
unnecessary words and maintains the authoritative
tone of the essay. Choices G and J ask questions and 35. The correct answer is D. A plural pronoun is required
would be appropriate only if the sentence ended with a to replace the antecedent discs, so them (choice D) is
question mark. Choice H changes the meaning of the correct. Choice B is singular and gender-specific and is
sentence. therefore incorrect here. Choice C is a plural pronoun
reserved for animate nouns, so would be incorrect here.
27. The correct answer is A. This sentence makes the point
that these foods are not indigenous to the nations that 36. The correct answer is J. This simple declarative
rely on them, and this point is crucial to the main idea of statement should end in a period, and no internal
the essay. Choice C makes this point awkwardly, while punctuation is needed between closer and look.
choices B and D do not make this point at all.
37. The correct answer is C. Willow tree is a common noun
28. The correct answer is H. The sentence begins with a that should not be capitalized unless it appears at the
series of subjects and should proceed to the verb beginning of a sentence, so choice C is correct.
without punctuation (choices F, G, and J) or unnec-
38. The correct answer is G. The underlined portion of the
essary words (choices F and J).
sentence contains an aside regarding salicylic acid and
29. The correct answer is D. For creates a smooth transition belongs in parentheses. The other answer choices are
from the previous sentence without changing the inappropriate treatments for this information.
meaning. Therefore (choice A) wrongly indicates that the
39. The correct answer is D. As written, the flow of
way we think of Italy is a direct consequence of the
information in the underlined portion of the sentence is
transformation of exchanged goods. Choices B and C do
confusing and awkward. Choice D avoids the confusion
not make grammatical sense in the context of this
by adjusting the word flow and is the correct answer.
sentence.
Choices B and C fail to eliminate the awkwardness and
30. The correct answer is F. This option emphasizes the confusion and are incorrect.
creative transformation of tomatoes and pasta by Italian
40. The correct answer is H. As written, the contraction
cooking. Choices G, H, and J do not express this crucial
we’ve is missing an apostrophe. Choice H correctly fixes
idea effectively.

  879
Peterson's ACT® Prep Guide 2018
Answer Keys and Explanations

the error. Choice G replaces we’ve with weave, which is a antecenent (women). Choice H combines the present
homonym and an inappropriate word choice here. and past tenses.
Choice J incorrectly replaces learned with leaned, which
47. The correct answer is B. As written, punctuation in the
is an inappropriate word choice here.
underlined portion of the sentence creates an awkward
41. The correct answer is B. Recognizing commonly used pause and is incorrect. Choice B correctly fixes the
phrases is a helpful skill to have for effective reading sentence. Choices C and D also contain incorrect
comprehension. The correct phrase for this sentence is internal sentence punctuation.
trial and error; the other choices sound similar, but are
48. The correct answer is H. This option maintains the
not appropriate phrases or word combinations for this
parallel structure set up by the preposition with, thus
sentence.
indicating that the two factors mentioned are sides of
42. The correct answer is J. The final sentence adds the same problem. As originally written, the phrase “not
relevant additional evidence supporting the paragraph’s to mention the fact” suggests an incidental piece of
topic that aspirin can have dangerous side effects, so information and not the explanatory information that
choice J is the best answer. The sentence provides actually follows the phrase. Choices G and J are
additional evidence; it does not repeat evidence already phrased awkwardly.
provided earlier in the paragraph as choice F suggests.
49. The correct answer is B. This option turns the fragment
While the paragraph has already provided some
(choice A) into a complete sentence and avoids the
evidence that aspirin can have dangerous side effects,
needless use of the passive voice in choice C. Choice D
additional evidence just strengthens the paragraph’s
is also a fragment.
thesis, so choice G is not the best answer. However, the
fact that aspirin can be hazardous to pregnant women 50. The correct answer is F. This option makes it clear that
or those who drink excessive amounts of alcohol is not the writer is being asked to write a piece about the First
necessarily more convincing evidence than any Lady’s charity work. Choices G and J change the
evidence previously given in the paragraph, so choice H sentence’s meaning. Choice H is unnecessarily wordy.
is not as strong an answer as choice J is.
51. The correct answer is B. The gerund phrase acts as a
43. The correct answer is C. Adding information about noun—in this case the object of the verb meant,
aspirin reducing the risk of heart attacks would best fit creating a correct sentence. The verbs in choices C and
in Paragraph 5, which covers the wide array of D are in the wrong form. The form should be positive
symptoms, inflammatory conditions, and illnesses that because the essay makes it clear that the author did
aspirins are used to help treat. break ground. As originally written, the sentence
contradicts its own meaning by including the word not.
44. The correct answer is F. An effective concluding
sentence ties up the ideas presented in the written 52. The correct answer is G. As written, the sentence
piece. The passage is about how aspirin, an everyday contains an incorrect word choice; the pronoun whom
object often taken for granted, is actually an interesting does not fit into the sentence as written. Choice G
bit of medicinal history and innovation, and choice F corrects the sentence: There was, however, an aspect of
effectively captures this notion. Choices G and J take the professional life for which many of us were not prepared.
focus off aspirin, and are incorrect. Choice H focuses on
53. The correct answer is B. The words “I remember” signal
a minor detail, and is too narrow in scope for a con-
us that the action in the sentence is a memory of
cluding sentence.
something that has already happened, so the past tense
45. The correct answer is C. This essay focuses solely on should be used in the underlined portion of the
aspirin and no other medicines, so it isn’t compre- sentence. Therefore, choice B is the correct answer.
hensive enough fulfill the requirements of an Choices C and D use the incorrect verb tenses.
assignment to research the history of medicines in the
54. The correct answer is F. The suggested paragraph
ancient world.
would add unnecessary details and distract from the
46. The correct answer is J. The phrase refers to the past, writer’s point. Since that suggested paragraph does not
so the verbs must be in the past tense. Choices F and G belong at any point in this essay, choices G, H, and J can
are in the present tense and choice G contains a all be eliminated.
singular verb (is) that does not agree with its plural

  880
www.petersons.com
Answer Keys and Explanations

55. The correct answer is C. This sentence is a question because illegible means unreadable. Choice B uses the
and should end in a question mark; the question word wrong verb form and the wrong word; ineligible means
what at the beginning of the sentence serves as a clue. “not entitled.”
Therefore, choice C is correct. Choice B incorrectly
62. The correct answer is J. The size of Haiti is completely
neglects to include end-of-sentence punctuation and is
irrelevant to the topic being discussed, so this under-
incorrect. Choice D incorrectly uses an em-dash, which
lined information should be deleted. Therefore, choice J
is not appropriate end-of-sentence punctuation.
is correct and choice F can be eliminated. Choices G and
56. The correct answer is J. This option avoids unnecessary H merely reword the irrelevant underlined phrase, so
wording and correctly conveys the writer’s feeling that both of those answer choices can be eliminated as well.
her complete honesty was inappropriate for her
63. The correct answer is D. The word Neighbors is part of
workplace. Choices F, G, and H would all leave the
a list and should be separated from the next item by a
sentence wordy and unclear.
comma. Choices A and B use incorrect punctuation, and
57. The correct answer is A. The sentence is correct as choice C uses no punctuation at all.
written and utilizes the correct form of the verb to share.
64. The correct answer is H. The willingness refers to that
The other answer choices utilize incorrect verb forms,
of the neighbors, friends, and relatives, so the pos-
given the context of the sentence.
sessive pronoun their is appropriate. Choices F and G
58. The correct answer is J. The opening sentence of the both use homonyms mistakenly: there means a
paragraph indicates that the writer has set up some particular spot, and they’re is a contraction of they and
kind of expectation that her work will not be first rate. are. Since they are does not make sense in this context
Therefore, in order for this paragraph to be most under any circumstances, choice J is incorrect.
effective, it should come at the end of the passage, after
65. The correct answer is D. The sentence describes the
the writer has shown us what she has done to set up
age of the author at the specified time, requiring the
such an expectation. In addition, the last line of the
simple past tense of the verb to be. Choice A would be
paragraph serves to point out the lesson that has been
used only if the subject did not get a chance to be
learned, concluding the essay. The paragraph would
something, which is not the case in this essay. Choice B
disrupt the flow of the essay if placed anywhere else, so
is a perfect gerund that indicates a state of being no
choices F, G, and H are wrong.
longer in effect, and it is not the clearest form to use in
59. The correct answer is A. In Paragraph 4, the writer sets this sentence. Choice C indicates an ongoing action that
up important, relevant information regarding her occurred in the past.
behavior that ultimately helped her to understand why
66. The correct answer is F. Because it logically follows the
she needed to put on a professional face. Choice B is
first sentence, the second sentence would not be as
incorrect because there are no historical details about
effective if moved elsewhere in the paragraph. It does
women in the workplace in the paragraph. The
not make sense to discuss the plot of Krik? Krak! before
paragraph mentions nothing about the writer’s male
the book is introduced in the first sentence, so choice G
colleagues (choice C) or friends (choice D) either.
is incorrect. It also does not make sense to introduce the
60. The correct answer is H. The writer is conveying an role of hardships in the plot after an example of such a
experience she had in her professional life that led to a hardship, so choice H does not make sense. Choice J
personal revelation. The paragraph is a personal essay, would split up the sentences that discuss the plot, so it
not a persuasive (choice F) or comparative (choice G) can be eliminated.
one. The writer’s personal integrity is not challenged in
67. The correct answer is C. No punctuation is needed
the essay, so choice J does not make sense.
before the prepositional phrase that begins with in, so
61. The correct answer is C. Illiterate means unable to read, choice C is correct and choice A can be eliminated. The
and population is a singular noun, so choice C is the sentence discusses plural families, so they must be
correct answer and choice D, which uses the plural verb seeking new lives, which eliminates choice D. An
are, can be eliminated. The original sentence is incorrect

  881
Peterson's ACT® Prep Guide 2018
Answer Keys and Explanations

apostrophe is used to show possession, yet the lives 72. The correct answer is H. Only one author is being
possess nothing in this sentence. Therefore, choice B discussed, and the possessive form is used to indicate
is wrong. that the subject matter belongs to the author. Choices F
and G indicate more than one author. Choice J lacks the
68. The correct answer is F. Since acclaim is a noun, it
elements needed to show possession.
should be modified by the adjective critical, not the
adverb critically (choice G) or the noun criticism (choice 73. The correct answer is D. This option is the most logical,
J). Adding too in front of this adjective changes the given the author’s subject matter. The proper negative
connotation, so choice H is not as strong an answer as construction is neither . . . nor. The constructions in
choice F. choices A and B create sentences that contradict their
own intended meanings. Choice C disrupts the proper
69. The correct answer is B. The first word that follows the
negative construction by following neither with or.
comma should be the subject of the modifying phrase
born in Port-au-Prince in 1969. That would be Danticat, 74. The correct answer is F. This option introduces the
which is not the first word in choices A, C, or D. Choice C informative, impersonal tone of the essay and provides
is also too wordy. an introduction to the theme of storytelling. Choice G is
too personal. Choice H is too vague. Choice J is too
70. The correct answer is J. This option links Danticat’s
informal and suggests the main theme of the essay is
childhood behavior with her development as a writer.
illiteracy, not storytelling.
Choices F and G do not create such a link. Choice H is
too vague. 75. The correct answer is C. The essay does not provide
nearly enough information about Haiti to serve as a
71. The correct answer is A. This option notes the
detailed history, so choice C is the best answer. Choices
similarity between Danticat and the heroine in the
A and B may be true in themselves, but neither makes a
clearest language. Like is the appropriate preposition to
convincing argument that this essay could serve as a
use when comparing nouns. The wordiness of choices B
detailed history of the hardships the people of Haiti
and D leaves the sentence unclear. Choice C suggests a
have endured. Choice D is simply untrue.
comparison between actions, not people.

  882
www.petersons.com
Answer Keys and Explanations

Mathematics
1.   E 16.   J 31.   C 46.   G
2.   J 17.   A 32.   F 47.   A
3.   C 18.   J 33.   E 48.   F
4.   K 19.   A 34.   J 49.   B
5.   E 20.   H 35.   C 50.   H
6.   F 21.   C 36.   J 51.   D
7.   E 22.   K 37.   A 52.   G
8.   H 23.   C 38.   G 53.   D
9.   C 24.   J 39.   D 54.   G
10.   F 25.   C 40.   K 55.   A
11.   D 26.   F 41.   D 56.   H
12.   H 27.   E 42.   H 57.   B
13.   D 28.   J 43.   E 58.   J
14.   J 29.   C 44.   F 59.   D
15.   C 30.   G 45.   B 60.   G

1. The correct answer is E. Plug the given numbers into Choice A is incorrect because you should be
the percent formula, where 32 is the part, 80 is the adding p and l, not multiplying them. In choice
whole, and x is the unknown percent: B, you should be adding px and lx, not multi-
32 x plying them. Choice D is incorrect because you
=
80 100 should be adding p and l, not multiplying them,
80 x = 3, 200 and you should multiply the resulting sum by x.
x = 40 Choice E is just the cost per hour—you must
multiply by the number of hours, x.
Choice A is the result of an arithmetic error; you are
off by a multiple of 10. Choices B and C are incorrect 4. The correct answer is K. This is the graph of y = 3x
because when taken out of 100, both 16 and 24 give shifted down 2 units. The shape remains the same
32 and the horizontal asymptote goes from y = 0 to y =
a number less than , so these percentages
80 –2. This results in the graph of choice K. Choice F is
incorrect because this is the graph of a log function;
cannot be correct. Choice D would imply that the
exponential functions do not have vertical asymp-
whole part being used is 100, but it is actually 80.
totes. Choice G is incorrect because this is the graph
2. The correct answer is J. The sum measure of a triangle’s of a log function; exponential functions do not have
interior angles is 180°, so the missing interior angle vertical asymptotes. Choice H is incorrect because
measures 180° – 40° – 55° = 85°. Since this angle the graph should increase from left to right. Choice
is supplementary to the angle measuring x°, J is incorrect because the horizontal asymptote
x = 180 – 85 = 95. should be y = –2.

Choice F is incorrect because the angles labeled x 5. The correct answer is E. Jacob uses a total of 7 ×
and 55 in the diagram are not congruent alternate 3 = 21 minutes, at a cost of $0.25 per minute, and
interior angles. Choices G and K are the likely result has to pay $1.00 for the card. 21 × 0.25 = $5.25, so
of an arithmetic error; you should be computing the total cost is $5.25 + $1.00 = $6.25.
180 – 85. Choice H is the measure of the third angle
Choice A only includes 1 of the 7 calls. Choice B
in the triangle, not the one labeled x.
would be the cost if each call were 1 minute
3. The correct answer is C. The machine costs p + l to long. Choice C does not include the $1 for the
operate each hour, so if it operates for x hours, the calling card. Choice D results from adding the $1
expression would be x(p + l). to 21 before multiplying by 0.25.

  883
Peterson's ACT® Prep Guide 2018
Answer Keys and Explanations

6. The correct answer is F. The profit before fees is $125 9. The correct answer is C. Substitute the values for w
– $85 = $40. The fee charged by the merchant is and z into the expression and simplify:
0.03($125) = $3.75. So, the total profit is $40 – $3.75 =
36.25 2 1 6 1 5
$36.25. Thus, the percent profit earned is × 100%. − −
85 3 9 = 9 9 = 9
Choice G is incorrect because you should divide by 85,  2 1  6 1  7
2 +  2 +  2 
 3 9  9 9  9
not 125. Choice H is incorrect because while the profit
earned before the fee is applied is $40, this does not 5
5 14 5 9 5
represent the percent profit. Choice J is incorrect = 9 = ÷ = ⋅ =
14 9 9 9 14 14
because you did not subtract the fee. Choice K is 9
incorrect because while the actual profit earned is
$36.25, this does not represent the percent profit. In choice A, you added fractions incorrectly; you
7. The correct answer is E. Convert the fractions to must first get a common denominator. Choice B has
the wrong sign. In choice D, you canceled terms, not
2 1
decimals: = 0.4 and = 0.125. Now add the three factors, common to the numerator and denominator.
5 8
Choice E is the reciprocal of the correct answer;
decimals: 0.4 + 0.125 + 0.177 = 0.702.
when converting a quotient of fractions to a product,
In choice A, you dropped 0.177 from the sum. In flip the fraction after the division sign.
1
choice B, you dropped from the sum. To get 10. The correct answer is F. The asymptote for such a
8
choice C, you forgot to carry one to the tenths place logarithmic function occurs at the value of x that makes
when adding. For choice D, you neglected to carry the input of the logarithm equal to zero. Here, that is x =
one to the hundredths place when adding. –4. Choice G is incorrect because this is a horizontal line,
and logarithmic functions do not have horizontal
8. The correct answer is H. The area of triangle ABC is 5
asymptotes. Choice H is incorrect because this is a
square inches. Plug this area and the length of the base,
horizontal line, and logarithmic functions do not have
BC , into the area formula to solve for the height, AB : horizontal asymptotes. Choice J is incorrect because this
value of x does not make the input of the logarithm
1
A = bh equal to zero. Choice K is incorrect because this value of
2
x does not make the input of the logarithm equal to
1
5 = (2)( AB ) zero.
2
AB = 5 2x
11. The correct answer is D. Write g(x) as y = . To find
x −1
the inverse, switch the x and y and solve for y:
Triangle ACD has the same height as triangle ABC. To
find the length of its base, CD, subtract BC from BD: 2x
y=
x −1
10 − 2 = 8. Therefore, the area of triangle ACD is
2y
1 x=
(8)(5) = 20. Choice F is the length of CD, not the y −1
2
x ( y − 1) = 2 y
area. In choice G, you used 8 in place of 10 as the
xy − x = 2 y
length of the base ABD. Choice J is the area of tri-
xy − 2 y = x
angle ABD. In choice K, you should have subtracted
y ( x − 2) = x
the area of triangle ABC from the area of triangle
x
ABD, not added it. y=
x −2
x
So, g −1( x ) = .
x −2

  884
www.petersons.com
Answer Keys and Explanations

12. The correct answer is H. When whole numbers are 15. The correct answer is C. When parallel lines are cut by
written as products of powers of prime numbers as is the a transversal, the alternate interior angles are equal.
case here—the least common multiple (LCM) is formed Therefore, ∠DCE measures 60°. Since the measures of
by using the highest power of each prime number that the angles in a triangle add up to 180°, ∠CED measures
occurs in any of the numbers. Applying that principle 180° – 40° – 60° = 80°.
here gives the LCM of p3q3r4. Choice F is the greatest
Choice A is incorrect because ∠CED is neither corre-
common factor. In choice G, you did not account for the
sponding nor alternate interior to ∠CDE, and so it is
third whole number in the list. Choice J is a common
not congruent to it in this scenario. In choice B, ∠CED
factor of all three whole numbers, but not a common
is neither corresponding nor alternate interior to
multiple. Choice K is a common multiple of all three
∠ABE, and so it is not congruent to it in this scenario.
whole numbers, but not the least common multiple.
For choice D, you must subtract this sum from 180
13. The correct answer is D. The difference between Colin’s degrees to get the desired angle. Choice E is not true
and Daryl’s average monthly salary is $500, so the since you can use the various relationships among
difference between their annual salaries is $500 × 12 = angles formed by two parallel lines cut by a
$6,000. We don’t know who earns more, but the transversal.
difference between their annual salaries is C − D dollars.
16. The correct answer is J. There are 8 different faces on
Thus, C − D = 6 , 000.
which the red die can land. For each of these faces,
Choice A is the difference of monthly salaries, not the there are 8 faces on which the blue die can come to a
annual salaries. Choice B is incorrect because you rest. Multiplying yields 8(8) = 64 different outcomes for
should be subtracting salaries, not adding them— the faces on which both die can come to rest in a single
even so, the right side would represent the difference roll. Choice F is incorrect because it is the number of
of monthly salaries, not annual salaries. In choice C, different outcomes for a single die. Choice G is incorrect
the average of the annual salaries is 6,000, which is because you should multiply the number of outcomes
not given in the problem. In choice E, you should be for each die, not add them. Choice H is incorrect
subtracting, not adding salaries. because it is half the correct number. Choice K is
incorrect because the order in which the dice are
14. The correct answer is J. Let x be the number of games
arranged does not affect the faces on which they come
Mike played. Then, Stan played 2x – 3 games. Together,
to rest. So the correct number should be one-half this.
they played x + (2x – 3) = 3x – 3 games. The cost of
playing this number of games is 0.25(3x – 3). Adding the 17. The correct answer is A. Rewrite the equation in
admission fee of 2($8.50) = $17, we get the equation slope-intercept form:
0.25(3x – 3) + 17 = 35. Solve for x:
2 x − 5 y − 1= 0
2 x − 1= 5y
0.25 (3 x − 3) + 17 = 35 2 1
y= x−
3 x − 3 + 68 = 140 5 5
3 x + 65 = 140 2 1
y= x−
3 x = 75 5 5
x = 25 2
So, the slope of the given line is . So, a line perpen-
5
The total number of games played is 3(25) – 3 = 72. 5
dicular to it must have slope − . Choices B, D, and E
2
Choice F is incorrect because you did not multiply the represent various errors based on using 2 as the
total number of games played by 0.25, the cost per slope of the given line; you must first solve for y
game. Choice G is incorrect because this is the before identifying the coefficient of x as the slope.
number of games Mike played. Choice H is incorrect Choice C would be the slope of a line parallel to the
because this is the number of games Stan played. given line.
Choice K is incorrect because this would imply that
18. The correct answer is J. Each of the larger walls has an
Stan played exactly twice the number of games that
area of 20 × 40 = 800 square feet, which requires 2
Mike did, which is not the case.
gallons of paint. Each of the smaller walls has an area of
1
20 × 30 = 600 square feet, which requires 1 gallons of
2

  885
Peterson's ACT® Prep Guide 2018
Answer Keys and Explanations

paint. The total area is 2,800 square feet, and the total In choices A and D, you did not apply the correct rule
1 1
number of gallons is 2 + 2 + 1 + 1 = 7. for simplifying quotients of powers of the same base;
2 2
Choice F is half of the correct answer, which may be the exponents should be subtracted, not multiplied
3
the result of only counting one of the smaller walls
or added. Choice B is incorrect because x 3 = x 2 ,
and one of the larger walls. Choice G results from
2
incorrectly calculating that each of the smaller walls not x 3 . In choice E, you did not compute the square
3
3
requires of a gallon of paint. Choice H may result root of a power correctly: x 3 = x 2 , not x 3 .
4
from a calculation error. Choice K is the number of 22. The correct answer is K. The total of the 6 original
gallons that would be needed if the total area were scores is (6)(80) = 480 points. The total of all 8 tests is (8)
24,000 square feet; 24,000 is the volume of the (85) = 680 points. Since the total of the 2 added tests
house, in cubic feet. must be 680 – 480 = 200, the average of these 2 tests is
19. The correct answer is A. Cross-multiply and then solve 200 ÷ 2 = 100. The values listed in choices F, G, H, and J
for w: will all result in an average lower than 85.
1
23. The correct answer is C. The area of a triangle is bh.
2w + 1 5 2
= We already know the height of the triangle, 9 centi-
3 6
(2w + 1)(6 ) = 5(3) meters. To find the base, use the Pythagorean theorem,
12w + 6 = 15 or recognize this as similar to a 3-4-5 right triangle:
12w = 9
9 3 9 2 + AC 2 = 15 2
w= =
12 4 81+ AC 2 = 225

Choice B is incorrect because this is the reciprocal of AC 2 = 144


the correct value of w. Choice C is incorrect because AC = 12
after cross-multiplying you did not apply the distrib-
utive property. Choice D is incorrect because you Since the base of the triangle is 12, the area of the
1
need to divide by 12, the coefficient of w, in the final triangle is (12)(9 ) = 54 square centimeters.
2
step of the solution. Choice E is incorrect because
Choice A is the length of the unknown side, in centi-
this is just the right side of the equation once you
meters. Choice B is the perimeter of the triangle, in
cross-multiplied. Now, solve for w.
centimeters. Choice D is double the area, which may
20. The correct answer is H. Set up a proportion between 1
result from forgetting the in the area formula.
similar sides: 2
24 x Choice E is the product of the two known sides,
=
30 40 which is not the area because side BC is neither the
30 x = 960 base nor the height of the triangle.
x = 32 1
24. The correct answer is J. Since the second file is 2
2
None of the values in choices F, G, J, and K retain the times as large as the first, multiply the amount of time
proportionality. 1
needed for the first file by 2 . You can do this either by
2
21. The correct answer is C. Use the exponent rules: converting 4 minutes and 26 seconds to seconds, then
converting back to minutes, or by multiplying both the
43 x 7 y 42 i 4 i x 3 1
2 4 3
= minutes and seconds by 2 , then combining the
3 x y 32 i y 2 2
results:
42 i 4i x3
=
32 i y2 1
4 minutes × 2 = 10 minutes
3 3 2
4 i 2 i x 2 8x 2
= = 1
3iy 3y 26 seconds × 2 = 65 seconds
2

  886
www.petersons.com
Answer Keys and Explanations

10 minutes and 65 seconds = 11 minutes and 5 28. The correct answer is J. First calculate the area of the
seconds 1
four triangles. Since the area of a triangle is bh, each
2
Choice F results from replacing 65 seconds with 5 1 3
triangle has an area of (a)(3a) = a 2 . Thus, the total
seconds, ignoring the additional minute. Choice G 2 2
1 3
results from only multiplying the minutes by 2 , not area of all four triangles is 4 × a 2 = 6a 2 . The easiest
2 2
the seconds. Choice H results from multiplying the
1 way to calculate the area of the shaded region is to
minutes by 2 and the seconds by 2. Choice K is
2 subtract the area of the four triangles from the area of
likely the result of an arithmetic error when con- square WXYZ, each side of which has a length of a + 3a
verting between seconds and minutes. = 4a. The area of square WXYZ is (4a)2 = 16a2, so the area
of the shaded region is 16a2 – 6a2 = 10a2. Therefore, the
25. The correct answer is C. The current paths require a
ratio of the area of the 4 triangles to the area of the
trip of 200 feet. A diagonal path would form an isosceles
shaded region is:
right triangle with the existing paths. By the rules of 6a 2 3
45°-45°-90° triangles, the length of the path would be =
10a 2 5
100 2 feet, or approximately 140 feet. The new path
would shorten the route by about 200 – 140 = 60 feet. In choice F, you compared the area of four triangles to
the area of the square WXYZ, not the shaded region.
Choice A is likely the result of an arithmetic error. For Choice G is incorrect because the area of the four tri-
choice B, you used the wrong triangle relationship; angles is 6a2, not 4a2. Choice H is incorrect because
this is a 45-45-90 triangle, not a 30-60-90 triangle. the area of the four triangles is 6a2, not 5a2. In choice
Choice D is the approximate length of the path. For K, the area of the shaded region is 10a2, not 8a2.
choice E, you used the wrong triangle relationship;
this is a 45-45-90 triangle, not a 30-60-90 triangle, 29. The correct answer is C. Get a common denominator
and this would be the length of the path. in the top and bottom separately; then, divide the
resulting fractions:
26. The correct answer is F. Let x be the original price.
5, 400 1 1 10 1 9
Then, 0.70x = 5,400, so that x = dollars. Choice G − −
0.70 10 100 = 100 100 = 100
1 1 5 1 4
is incorrect because you should divide by 0.7, not − −
5 25 25 25 25
multiply by it. This represents 70% of 5,400. 9 4 9
25
Choice H is incorrect because this is 30% of 5,400. = ÷ = 9
i 4
=
100 25 4 100 16
Choice J is incorrect because you do not apply per-
Choice A is the result of multiplying the numerator
centages by adding the percent to a quantity. Choice K
and denominator of the complex fraction instead of
is incorrect because the denominator should be 0.7.
dividing them. In choice B you canceled terms, not
27. The correct answer is E. Each wall is 100 yards long and factors, common to the numerator and denominator.
15 yards high. Convert these yards to feet, since the Choice D is the reciprocal of the correct answer.
paint coverage is measured in square feet. There are 3 Choice E is the result of flipping the fraction before
feet in a yard, so each wall measures 300 feet by 45 feet, the division sign when converting the quotient of
and the total square footage is 2 × 300 × 45 = 27,000 two fractions to a product.
square feet. One gallon of paint covers an area of 150
30. The correct answer is G. Use the two labeled points to
square feet, so calculate the number of gallons needed
determine the slope of the line:
for this job:
−3 − ( −1) 2 1
m= =− =−
1 gallon 4 − ( −2) 6 3
27, 000 square feet × = 180
150 square feet
Using point-slope form with the point (–2, –1) yields
the equation
In choice A, you did not convert to feet when com- 1
puting square footage. In choice B, you did not y − ( −1) = − ( x + 2)
3
convert 100 yards to feet. In choice C, you did not 1 2
y + 1= − x −
account for the fact that there are two walls. Choice D 3 3
is the information given to you; you must determine 1 5
y=− x−
total square footage and then divide by 150. 3 3
  887
Peterson's ACT® Prep Guide 2018
Answer Keys and Explanations

Choice F is incorrect because the slope is the change 33. The correct answer is E. If the average of x numbers is
in y divided by the change in x. Choice H is incorrect 15, then the total of these numbers is 15x. When two
because the slope is the change in y divided by the numbers are increased by y, 2y will be added to this
change in x, and you seemed to have eyeballed the total. The value of x does not change, so the new
y-intercept. Choice J is incorrect because you seemed 15 x + 2 y 2y
average will be = 15 + . This is an increase
to have eyeballed the y-intercept. Choice K is incorrect x x
because the sign of the constant term should be 2y
of over the original average.
negative. x

31. The correct answer is C. Construct a frequency table: Choice A is the amount added to the overall total
before averaging. Choice B is just the amount by
Data value Frequency which each of the two numbers increased, not the
0 3 amount by which the average increases. Choice C is
1 9 the reciprocal of the correct answer. The value given
2 3 in choice D should be doubled, since two numbers
were increased by y.
3 1
4 4 34. The correct answer is J. The unshaded sectors AOB and
5 5 2 1 11
BOC comprise a total of + = of the circle.
5 3 15
6 0
This means that the shaded region, sector AOC, takes up
7 2 4
the remaining of the circle. A circle has 360 total
8 3 15
degrees, so set up a proportion to find the measure of
The mode is the most frequently occurring data value,
∠AOC:
which is 1. The median is the average of the 15th and
2+2 4 m∠AOC
16th data value, namely = 2. The mean is the =
2 15 360°
average of all the data values: 15(m∠AOC ) = 1, 440°
m∠AOC = 96°
0(3) + 1(9 ) + 2(3) + 3(1) + 4( 4 ) + 5(5) + 6(0 ) + 7(2) + 8(3)
30 Choice F is incorrect because the number of degrees
97 7 in a circle is 360, not 180. In choice G, you seem to
= =3 . 1
30 30 have mistakenly said that the shaded region is of
4
the circle—don’t eyeball it! Choice H is close, but is
So mode < median < mean.
likely the result of eyeballing the region rather than
using the information provided in the problem.
32. The correct answer is F. First, find cosq using the
Choice K is the measure of 360 – ∠AOC, not just
identity sin2 q + cos2 q = 1 :
∠AOC.

sin2 q + cos2 q = 1 35. The correct answer is C. The ladder forms a right
 2
2
triangle with the house and the ground, so use the
  + cos q = 1
2

3 Pythagorean theorem to solve for the distance:


4 10 2 + b 2 = 12 2
+ cos2 q = 1
9
b 2 = 144 − 100
5
cos2 q =
9 b = 44
5 5 ≈ 6.6
cos q = ± =±
9 3 In choice A, when using the Pythagorean theorem,
you forgot to square the legs and hypotenuse. For
Since the terminal side of q is in the second quadrant,
choice B, you computed the squares of the legs and
5 hypotenuse incorrectly; note that a2 does not equal

5 cos q 3 5
use cos q = − . Now, cot q = = =− . 2a. For choice D, you treated the hypotenuse as a leg
3 sinq 2 2
3 in the Pythagorean theorem. For choice E, you forgot
to take the square root.

  888
www.petersons.com
Answer Keys and Explanations

36. The correct answer is J. Since the diameter of the circle 40. The correct answer is K. Calculate as follows:
is 12 yards, the radius must be 6 yards. So, the area of
 4 2
the circle is 62π = 36π square yards. In choice F, you h  −  = 1−
 x  4
incorrectly squared π when computing the area. In 3 − 
 x
choice G, you used the wrong formula for area; it should 2  x
be πr2, where r is the radius. Choice H is the circum- = 1− = 1+ 2  
12  12 

ference of the circle. In choice K, you used the diameter x
instead of the radius when computing the area. x 6+ x x +6
= 1+ = =
6 6 6
37. The correct answer is A. There are 3(24) = 72 tea-
spoons in the jar of cough medicine. Divide the total x+6 x+ 6
Choice F is incorrect because ≠ . Choice
$4.50 6 6
cost by 72 to get the cost per teaspoon: ≈ $0.06.
72 G is incorrect because you did not simplify the
Choice B is incorrect because you divided by 30 instead complex fraction correctly. Choice H is incorrect
of 72, the number of teaspoons in the jar. Choice C is 2x 2x +1
because + 1≠ ; you must first get a
incorrect because you divided by 24; this is the cost per 12 12
tablespoon, not teaspoon. Choice D is incorrect because
common denominator. Choice J is incorrect because
you are off by a factor of 10. Choice E is incorrect
because you divided by 3, not 3(24). 4
it is the product of h(x) and − .
x
38. The correct answer is G. The given line is vertical since
it passes through two points having the same x-coor- 41. The correct answer is D. Use the second portion of the
dinate. So, the desired line must be horizontal. Since it function since that is the interval to which x = 0 belongs.
passes through the point (–1, 4), its equation must be Doing so yields F(0) = 4. Choice A is incorrect because
y = 4. In choices F, H, and K, you used the point (–1, 4) the portion of the function corresponding to the
when computing the slope of the given line, but it does interval containing zero is, in fact, defined at 0; so, the
not pass through this point. Choice J is parallel to the y-intercept does exist. Choice B is incorrect because this
given line. is the x-intercept. Choice C is incorrect because you
used the wrong portion of the function; 0 does not
39. The correct answer is D. Let x be the missing leg of the
belong to the interval for which the function is defined
triangle. Using the Pythagorean theorem yields 22 + x2
by the third expression. Choice E is incorrect because
= 52, so that x2 = 21. Thus, x = 21. The area of the you used the wrong portion of the function; 0 does not
1
( )
triangle is therefore (2) 21 = 21 square yards. In
2
belong to the interval for which the function is defined
by the first expression.
choice A, you did not square the lengths of the sides
when applying the Pythagorean theorem. In choices B 42. The correct answer is H. Three pairs of prime numbers
and C, you applied the Pythagorean theorem treating add to 24: 5 + 19, 7 + 17, and 11 + 13. The smallest
the hypotenuse as a leg. In choice E, you used the resulting product is 5 × 19 = 95, so 95 is the least
hypotenuse when computing the area, but you should possible value of xy.
use both legs. Choice F gives you a prime, so it cannot be a product
of prime numbers. You get choice G from the choice
2 + 22, but 22 is not prime. You get choice J from 7
and 17 and choice K from 11 and 13, but neither one
yields the least product.

  889
Peterson's ACT® Prep Guide 2018
Answer Keys and Explanations

43. The correct answer is E. The radius is the length of the ( )( )


The area of the rectangle is 4 2 8 2 = 64.
segment connecting the two given points:
For choice F, note that 2 • 2 = 2, not 2.
(2 − ( −4 ))2 + ( −2 − 3)2 = 62 + 52 = 36 + 25 = 61.
Choice H is the perimeter, not the area. Choice J is
Using the center (h, k) = (–4, 3) and radius r = the square of the length of a side of the rectangle.
61 in the standard form for the equation of For choice K, you seem to have assumed the rect-
a circle (x – h)2 + (y – k)2 = r2 yields the equation angle was a square with side 12. You must use the
( 61)
2
( x − ( −4 ))2 + ( y − 3)2 = , which simplifies to distance formula to find a side length.

( x + 4 ) + ( y − 3) = 61 . Choices A and B have the


2 2
47. The correct answer is A. In order for the expression to
signs of the center wrong, and the right side in be positive, x and z must either both be negative or
choice B should be squared. In choice C, you did not both be positive. Choice B would produce a quotient
square the differences in the x- and y-coordinates involving an odd number of negative signs, which must
when computing the distance between the given be odd. Choice C is incorrect because option II is
points. In choice D, you used the wrong point for the false—this would produce a quotient involving an odd
center and should square the right side. number of negative signs, which must be odd. Choices
D and E are incorrect because options II and III are false.
44. The correct answer is F. Solve for z:
Option II is false because this would produce a quotient
involving an odd number of negative signs; option III is
0.3 z − 0.24 = 1.2 (0.3 − z ) false because if x = 0, then the quotient is 0.
0.3 z − 0.24 = 0.36 − 1.2 z
1.5 z = 0.60 1 1
48. The correct answer is F. Since = = 5−4 , it follows
0.60 625 54
z= = 0.4
1.5  1 
that log5 
 625 
( )
= log5 5 − 4 = −4. Choice G is incorrect
0.60
z= = 0.4 1
1.5 because 5 − 3 = . In choice H, you applied the power
125
1
So z i 10 −3 = (0.4 ) i 10 −3 = 0.0004 . All other choices
in the wrong way: , but 625 4 = 5. Choice J has the
are off by various powers of 10.
45. The correct answer is B. Tangent equals opposite wrong sign. In choice K, you incorrectly multiplied the
divided by adjacent. In an isosceles right triangle, the base times the power.
legs are equal. In this case, that means the opposite and
49. The correct answer is B. For x to have an inverse
adjacent sides are equal, so tan x° = 1. Choice A is either
relationship with y, it must be in the denominator of the
sine or cosine of 45 degrees. Choice C is either the
right side. For z to have a direct relationship with y, it
secant or cosecant of 45 degrees. For choices D and E,
must be in the numerator of the right side. Only the
you used the wrong triangle relationships—this is a z
45-45-90 triangle, not a 30-60-90 triangle. equation y = does this.
x
46. The correct answer is G. Solve for the width and length
of the rectangle using the distance formula: Choice A is incorrect because the right side should
be the reciprocal of what is listed here. Choice C is
incorrect because it means y is inversely proportional
w = ( −5 − ( −1)) 2 + ( −2 − ( −6 )) 2
to both z and x. For choice D, applying the logarithm
= 16 + 16 rules, this would make log(y) inversely proportional
= 32 to z and proportional to log(x). For choice E, applying
=4 2 the logarithm rules would make log(y) inversely pro-
portional to x and proportional to log(z).

l = ( −1− 7) 2 + ( −6 − 2) 2
= 64 + 64
= 128
=8 2

  890
www.petersons.com
Answer Keys and Explanations

50. The correct answer is H. Divide 114 feet into 3-foot 54. The correct answer is G. Begin by plugging in 100 for x.
segments to see that there are 38 such segments. The If 100 is increased by 20 percent, the result is 120. This
cost, before tax, is therefore 38($8.10) = $307.80. The 5% number is then decreased by 30 percent. Thirty percent
sales tax on this amount is 0.05($307.80) = $15.39. So, of 120 is 36, so the final result is 120 − 36 = 84. This is
the total cost is $307.80 + $15.39 = $323.19. the same as decreasing 100 by 16 percent. Choice F is
Choice F is incorrect because you subtracted 5% of the incorrect because the percentages are actually acting in
pre-tax cost instead of adding it to it. Choice G is different directions, so you cannot simply add them in
incorrect because you did not include the 5% tax. this manner. Choices H and J are incorrect because you
Choice J is incorrect because you used $8.10 as the cost cannot just add percentages; they must be applied to a
per foot, not per 3-foot segment, and did not include whole. For choice K, you assumed that x was initially
the 5% tax. Choice K is incorrect because you used $8.10 decreased by 20% and then applied 30% to 0.80x.
as the cost per foot, not per 3-foot segment.
55. The correct answer is A. As shown in the diagram
51. The correct answer is D. Solve for y: below, the height of the pole, x, is opposite a 70° angle,
and the hypotenuse of the triangle is 50.
3(1− 2 y ) 2 − 4 y
− =y
4 3
12(1− 2 y ) − 4(2 − 4 y ) = 12 y
12 − 24 y − 8 + 16 y = 12 y
4 − 8 y = 12 y
4 = 20 y
1
y= 50
5 x
−1
 1
So, y −1 =   = 5. In choices A, C, and E, you solved
 5
an equation of the form ay = b incorrectly (you should
divide both sides by a) and computed y–1 incorrectly. 70º
In choice B, you multiplied the base and exponent when
computing y–1.
Sine equals opposite divided by hypotenuse:
52. The correct answer is G. The period of a graph of the
2p x
form y = Acos(Bx + C) + D is . So, the period of the = sin 70°
B 50
 p 2p x = 50 sin 70°
graph of y = 4 cos  3 x −  + 1 is . Choice F is the
 2 3
phase shift. In choice H, you incorrectly identified the Choice B is the length of the unlabeled leg of the tri-
coefficient of x as the period. Choice J is the amplitude. angle. In choice C, you used an incorrect definition of
In choice K, you did not divide by the coefficient of x. tangent. In choices D and E, you used an incorrect
definition of cosine.
53. The correct answer is D. Let s be a side of the square.
Two adjacent sides and the corresponding diagonal 56. The correct answer is H. The inequality 2|3x + 1| > 8 is
form a right triangle. So, using the Pythagorean equivalent to |3x + 1| > 4. This implies that either 3x + 1
theorem yields s2 + s2 = 82, or equivalently, 2s2 = 64. So > 4 or 3x + 1 < –4. Solving the first inequality yields x > 1
s2 = 32 and hence, s = 4 2 inches. Therefore, the 5
and solving the second one yields x < − . So, the
( )
perimeter is 4 4 2 = 16 2 inches. Choice A is the
5
3
length of a side of the square. Choice B is one-half the solution set is {x: x > 1 or x < − }. In choices F and J,
3
perimeter. In choice C you did not use the Pythagorean you multiplied both sides of the original inequality by 2
theorem correctly to find the length of a side of the instead of dividing by 2. Choices F, G, and K only provide
square. Choice E is the area of the square. one of the two rays that comprise the solution set.

  891
Peterson's ACT® Prep Guide 2018
Answer Keys and Explanations

57. The correct answer is B. For the lines to never intersect, 59. The correct answer is D. The outcomes that satisfy the
they must be parallel. So, they must have the same given criteria are as follows:
slope. The equation 2x – cy = 1 can be written equiva-
1 3 5 7 9 11 13 15 17 19 21 23 25 27 29 and 10 20 30
2 1 2
lently as y = x − . So its slope is . The slope of the So, 18 of the possible 30 outcomes satisfy the condition.
c c c
second line is 8c. Equating the slopes yields the 18
Thus, the probability is = 0.60. Choice A is wrong
2 30
equation = 8c . This is equivalent to 2 = 8c2, so that because while 18 outcomes satisfy the condition, you
c must divide this by the number of outcomes possible,
1 1 1
c = . Solving for c gives c = − , c = . Choice A
2
not by 100. Choice B is the probability of NOT selecting
4 2 2
would produce lines that are perpendicular. Choice C such a card. Choice C does not account for the cards
10, 20, or 30. In choice E, you double counted the out-
shows the reciprocals of the correct values of c. Choice comes 5, 15, and 25.
D is the result of solving an equation of the form ax = b
60. The correct answer is G. A shirt and two ties cost $105,
incorrectly; you should divide both sides by a, not
which translates to s + 2t = 105. Two shirts and one tie
multiply both sides by it. In choice E, you squared both
cost $135, which translates to 2s + t = 135. Multiply both
1 sides of the first equation by 2: 2s + 4t = 210. Then
sides when attempting to solve c2 = instead of taking
4 subtract the second equation from the first equation to
the square root of both sides.
solve for t, the price of one tie:
58. The correct answer is J. Find the slope of the given line.
5−2 2 s + 4t = 210
= 3 . Use the point-slope formula to find the
−1+ 2 −(2 s + t = 135)
equation of the line.
3t = 75
y − y1 = m( x − x1 ) t = 25
y − 5 = 3( x + 1)
In choice F, when solving the system you added the
y − 5 = 3x + 3
t-terms but should have subtracted them. In choice
y = 3x + 8
H, you likely made an arithmetic error. In choice J,
The equation for line m is y = 3x + 8. Since line n is you have the cost of two ties, not one. In choice K,
1 you calculated the cost of one shirt.
perpendicular to line m, it must have a slope of − .
3
By putting the given equations in slope-intercept
form, you can see that only the line represented by
1 1 14
3y = –x + 14 has a slope of − : y = − x + . The
3 3 3
1
slopes of the lines in choices F, G, and H are not − .
3
Remember, the product of the slopes of perpen-
dicular lines must be –1. Choice K is incorrect
because the slope of line m is 3. Since the product of
the slopes of perpendicular lines must be –1, this
cannot be the equation of line n.

  892
www.petersons.com
Answer Keys and Explanations

Reading
1.   D 11.   B 21.   C 31.   C
2.   H 12.   J 22.   F 32.   J
3.   B 13.   B 23.   D 33.   C
4.   G 14.   H 24.   H 34.   F
5.   B 15.   A 25.   A 35.   D
6.   J 16.   F 26.   H 36.   H
7.   C 17.   D 27.   C 37.   A
8.   F 18.   H 28.   F 38.   H
9.   D 19.   D 29.   D 39.   D
10.   H 20.   H 30.   G 40.   G

1. The correct answer is D. Paragraph 1 states that Mr. neither word is used to describe Miss Brooke, so
Bennet addressed his second daughter, and the you can eliminate choices H and J. However, both
following line finds him addressing Lizzy, so choice D is Celia and Sir James are familiar with the notion that
correct and choice B can be eliminated. A second Miss Brooke is too religious, so choice G is the best
daughter could be the youngest one if there are only answer.
two daughters, but we know based on this passage that
5. The correct answer is B. The main purpose of lines
there are at least three daughters in the Bennet family
28–30 is to reveal Sir James’ inner thoughts about
(Lizzy, Kitty, and Mary), so choice A is not the correct
Miss Brooke, so choice B is the best answer. This is
answer. The only mention of anyone being sensible in
not a quotation, though, so choice A cannot be
this passage refers to Mary, not Lizzy, and she fails in her
correct. The information in lines 28–30 does not
attempt to do so, so choice C is not the correct answer.
contradict the idea that Sir James is charming, so
2. The correct answer is H. Mrs. Bennet uses this choice C is incorrect. While lines 28–30 do reveal
figurative phrase in reference to her nerves and to some of Miss Brooke’s qualities that Sir James loved,
indicate that Kitty’s coughing is making her upset, so the lines’ main purpose is not structured as a list.
choice H is the best answer. While physically tearing up
6. The correct answer is J. The sentence reads: “Celia
something is destructive, and perhaps even violent,
felt that this was a pity.” It is clear from this sentence
Kitty is not literally tearing up Mrs. Bennet’s nerves, so
that the word pity is a noun, so you can eliminate
choices F and G are not correct. Mrs. Bennet’s issue is
choice F since sorry can be used only as an
emotional, not physical, so choice J is not the best
adjective. Compassion (choice G) and sympathy
answer.
(choice H) have the same meaning, so they most
3. The correct answer is B. After Mr. Bennet makes the likely cancel each other out. They also would not
suggestion, Mrs. Bennet dismisses it as a breach of make sense if used in place of pity in the original
etiquette, which is apparently very important in this sentence. The only answer choice that would
story. According to etiquette, Mrs. Bennet cannot make is shame.
the introduction when she does not know Mr. Bingley
7. The correct answer is C. In Passage A, Mrs. Bennet
personally. Although Mrs. Bennet believes Mr. Bennet
has little patience for her daughters’ comments,
has made a poor suggestion, it is extreme to conclude
and Miss Brooke is abrupt and cold with Sir James,
that she thinks he has lost his mind (choice A), he does
so the answer that best describes both women is
not understand her (choice C), or he does not love her
short-tempered (choice C). Demure means shy, and
(choice D).
neither woman has any reservations about
4. The correct answer is G. Although Miss Brooke’s expressing her opinions, so choice A does not make
behavior toward Sir James may be interpreted as rude, sense. Disgusted may describe Miss Brooke since
no one in the passage ever says she is too rude, so she may “despise” Sir James, but it is too harsh a
choice F is not the best answer. Although the words word to describe Mrs. Bennet, so choice B is not the
happy and scruples are used in the first paragraph, best answer. The narrator of Passage B describes

  893
Peterson's ACT® Prep Guide 2018
Answer Keys and Explanations

Miss Brooke’s behavior as an amusing contrast to Sir that organized a protest march (choice C). While at least
James’s solicitous amiability, but that word does not two women had voted successfully in the Washington
really describe her in general, and it does not really Territory, there is no implication that the women in
describe Mrs. Bennet either, so choice D is not the best question were able to vote in that area after being
answer. refused in the District of Columbia, so choice D is
incorrect.
8. The correct answer is F. This is an inference question.
Kitty becomes fretful when her parents interpret her 12. The correct answer is J. In this passage, Susan B.
coughing as impudence. This implies that she is Anthony first successfully secures the right to vote by
concerned about how others interpret her behavior. invoking the Fourteenth Amendment of the US
However, Miss Brooke says exactly what is on her mind Constitution, so choice J is the best answer. While
no matter how others might interpret her words. So Anthony read a publication called The Revolution and
choice F describes a difference between the two hoped for more militancy among women, such matters
characters. Choice G describes how Miss Brooke is do not necessarily support the conclusion in choice F,
different from Kitty, not the other way around. Choice H since the passage never indicates Anthony took violent
is Sir James’s interpretation of Miss Brooke’s behavior; it measures to secure voting rights for women. While
is not anyone’s interpretation of Kitty’s. Mrs. Bennet Anthony did take her issue to the press, this was not the
describes Ms. Long as selfish and hypocritical; no one action that was most successful for her cause, so choice
describes Kitty this way, so choice J is wrong. G is not as strong of an answer as choice J. Her actions
described in this passage are to secure the right to vote,
9. The correct answer is D. At the end of Passage A, the
but voting, itself, is not one of those actions, so choice H
narrator reveals Mary’s unspoken wish “to say some-
is not as strong of an answer as choice J.
thing sensible.” The first paragraph of Passage B reveals
Celia’s private thoughts. Choice D is the best answer. 13. The correct answer is B. The third paragraph describes
Kitty’s inner thoughts are never revealed in Passage A, numerous other women who attempted to vote or
so choice A is incorrect. Neither Elizabeth nor Miss secure the right to vote in Susan B. Anthony’s time, so
Brooke’s inner thoughts are known in these passages, so choice B is the best answer. The beginning of the
choice B is wrong. Mr. Bingley never even appears in paragraph specifically states that the actions it
Passage A, so choice C cannot be correct. describes only gave Anthony hope that more successful
militancy was to come, which suggests that a militant
10. The correct answer is H. In Passage A, the Bennets
effort to secure women the right to vote was not yet in
intend to introduce their daughter Lizzy to Mr. Bingley
full force. Therefore, choice A can be eliminated. The
in the hope that there will be a romantic connection
paragraph refers to several specific women who were
between them. In Passage B, Sir James expresses
working to secure the right to vote, but it does not
romantic interest in Miss Brooke. Therefore, you can
support the extreme statement in choice C. The
infer that both men are unmarried. The other answer
paragraph also refers to the fact that some women took
choices are difficult to confirm. Mr. Bingley does not
their cases to the Supreme Court, but it does not quite
actually appear in Passage A, so it is not possible to
support the conclusion in choice D, since those cases
know whether or not he is extremely charming (choice
were unsuccessful.
F), uninterested in etiquette (choice G), or if he even
meets Lizzy (choice J). 14. The correct answer is H. While Susan B. Anthony is the
main focus of this passage, the passage describes the
11. The correct answer is B. The answer to this question
actions of many people who worked toward enacting
can be found in the second paragraph, which states,
social change, so choice H is the best answer. Choice F
“Sarah Andrews Spencer, Sarah E. Webster, and seventy
would have been correct if the passage discussed only
other women marched to the polls to register and vote
Anthony. The passage focuses on securing the right to
in the District of Columbia. Their ballots refused, they
vote, but using voting as a tool to enact social change is
brought suit against the Board of Election Inspectors,
not discussed in the passage, so there is no support for
carrying the case unsuccessfully to the Supreme Court
choice G. Choice J is too specific, since taking cases to
of the United States.” This supports choice B. Choice A
the Supreme Court was just one of the numerous
confuses the situation in the District of Columbia with
methods for affecting change discussed in the passage.
one in Detroit. While the passage states that the women
marched to the polls to register and vote, it does not say

  894
www.petersons.com
Answer Keys and Explanations

15. The correct answer is A. The author indicates that it 19. The correct answer is D. According to the ninth
was Susan B. Anthony’s persistence that caused Jones paragraph, the Chicago Tribune suggested that Anthony
and Marsh to consent to allowing the women to vote. should hold public office, which indicates that the paper
While she did read lines from the Fourteenth supported her, rather than criticized her. Therefore,
Amendment, the author does not imply that this is what choice D is the best answer. The New York Graphic
inspired Jones and Marsh to consent. There is a (choice A), however, contained editorial cartoons that
suggestion that Anthony stirred up an argument and “caricatured her unmercifully.” And since the paragraph
that Jones and Marsh broke from their colleagues when points out that such critical cartoons were very
they decided to consent to allowing Anthony and the common, choice C is not the best answer either. The
other women, but there is not enough evidence to Rochester Union and Advertiser criticized Anthony as
support the very specific conclusion in choice C. lawless and charged that her actions “. . . proved women
Anthony only rounded up more women to vote after unfit for the ballot.”
Jones and Marsh allowed them the right to vote, so
20. The correct answer is H. The first line of the eighth
choice D is incorrect.
paragraph states that “Election day did not bring the
16. The correct answer is F. Choice F, relevant, is the general uprising of women for which Susan had hoped,”
answer choice that would make the most sense if used which supports the conclusion in choice H. Choice F is
in place of pertinent in line 61. While interesting not the best answer, since Lillie Devereux Blake and
(choice G) would make sense, it does not fit the specific Matilda Joslyn Gage only tried to vote; they did not do
meaning of line 61 as well as choice F. Persistent (choice so successfully. Choice G is not a strong answer either,
H) may be spelled similarly to pertinent, but the two since Stanton only seemed to lack motivation; there is
words have very different meanings. Choice J seems to no evidence in this passage that she actually criticized
confuse pertinent for penitent, the latter of which means Anthony. Choice J is wrong because it is too extreme;
“sorry or remorseful.” while some news media outlets did mock Anthony, the
ninth paragraph proves that not all news media outlets
17. The correct answer is D. According to the seventh
mocked her.
paragraph, the news media reported on the registration
of women voters in Rochester after Susan B. Anthony 21. The correct answer is C. Each answer choice can be
had successfully gotten sixteen women to register to used as a synonym for inspired, but choice C makes the
vote (choice D). Choice A describes something that most sense in this particular context. A single painted
most likely happened after the news media reported canvass cannot really be animated, so choice A does not
Anthony’s accomplishment. Choice B describes make much sense. Encouraged does not make sense in
something that definitely happened after it. Choice C this context either, so choice B can also be eliminated.
describes something that Anthony only says will There is no real indication that education is what
happen in the future in this passage, so the passage inspired Monticelli to use color in a particular way, so
does not even make clear whether Post went through choice D is not the best answer, either.
with her plans to sue the registrars.
22. The correct answer is F. The author clearly appreciates
18. The correct answer is H. While the passage is not Impressionism, and in the third paragraph, he writes
specifically about the difficulties women faced in that Impressionism rejects symbols. Therefore, choice F
becoming lawyers during Susan B. Anthony’s time, it is is a logical conclusion. The author indicates that
about difficulties women faced during that period, and Impressionism is different from Romanticism and
the details about Myra Bradwell are relevant to that Classicism, but that does not mean he would necessarily
particular theme, so choice H is the best answer. Myra agree with the statement in choice G. The author states
Bradwell was denied admission to the Illinois bar that Impressionistic art is “aloof from mythology,” which
because she was a woman, not because she was implies the opposite of choice H. The author implies
unqualified, so choice F is incorrect. There is no that Impressionists were technically advanced, but
exaggeration in this passage; women really did face nothing he writes really supports the extreme con-
major challenges during Susan B. Anthony’s time, so clusion in choice J.
choice G is a weak answer. While it is likely that Myra
23. The correct answer is D. The author compares
Bradwell was bright, choice J does not explain why the
Monticelli’s use of color to a musical theme to indicate
author discusses her difficulties in the third paragraph.
that such basic elements of art and music can still have

  895
Peterson's ACT® Prep Guide 2018
Answer Keys and Explanations

great emotional impact, so choice D is the best answer. describes the first two paragraphs, so it is not a very
Choices A and C take the comparison between complete explanation of the primary purpose of the
Monticelli’s use of color and musical themes too literally. passage as a whole. Choices B and D each reflect
There is no evidence that Monticelli actually used music specific details in the passage and both fail to capture
as an inspiration for his art (choice A) or that he actually the purpose of the entire passage.
composed music (choice C). Choice B is too extreme;
28. The correct answer is F. The author describes
while Monticelli may have emphasized one particular
Monticelli’s brush as “full, fat and rich,” which supports
color in each of his paintings, there is no evidence that
statement I. However, the author only compares
he never used more than one color in his paintings.
Monticelli’s reliefs to jewels and his construction of color
24. The correct answer is H. In the third paragraph, the to musical themes; he does not actually suggest that
author writes that the Impressionists occupy them- Monticelli painted jewels (statement II) or was directly
selves with consideration of light (statement II) and influenced by music (statement III). Therefore, the best
picturesqueness (statement III), but have an antipathy answer should include statement I and eliminate
to abstraction, which means that they rejected abstract statements II and II, and only choice F accomplishes this.
imagery (statement I). Therefore, the correct answer Choice G includes one true statement and one false
must include statements II and II and eliminate one. Choice H includes only the two false statements.
statement I, and that is what choice H does. Choice F Choice J includes the one true statement as well as the
only includes the false statement. Choice G includes two false ones.
only one of the two true statements. Choice J includes
29. The correct answer is D. A fact is a statement with
both the false statement and the two true ones.
which no one can argue, and this can be said only of
25. The correct answer is A. The author’s use of strong, choice D. The author states choice A in the second
emotional words such as delight, astonishment, and joy paragraph, but it is only an opinion, since not everyone
support the inference in choice A. While the author may agree that Monticelli’s use of color was inspired.
states that Monticelli has an entirely personal tech- Choice B is not a fact, since the author compares
nique, he immediately follows this by stating that Monticelli to both Watteau and Turner, which implies
Monitcelli’s work can be “compared with that of Turner,” that Monticelli’s style was not actually entirely personal.
so choice B is not the best answer. The author explains Not everyone may agree with the author’s statement
in the paragraph that Monticelli painted landscapes, so that “Every picture by Monticelli provokes aston-
choice C is incorrect. Choice D is a conclusion that can ishment,” so choice C is incorrect.
be reached after reading the first paragraph, not the
30. The correct answer is G. Choice G is the best interpre-
second one.
tation of this line, especially since the author empha-
26. The correct answer is H. In the fourth paragraph, the sizes the role of spontaneity in creating Impressionistic
author writes, “Manet, who, by his spirit and by the art. While the phrase does imply that Impressionists
chance of his friendships, grouped around him the were reluctant to discuss their art, it does not support
principal members, commenced by being classed in the the extreme conclusion in choice F. Since the author
ranks of the Realists . . .,” which supports choice H. The discusses the role of spontaneity in impressionism,
author does not offer choice F as an explanation and choice H is not true. Choice J is a statement the author
never implies that Impressionist art does not depict makes in the paragraph, but it has nothing to do with
contemporary scenes, so choice G is incorrect. While the phrase in question.
their names are similar, the author never implies that
31. The correct answer is C. In the third paragraph, the
Manet and Monet were confused with each other, nor
author explains that “The form of a coast, the configu-
does the author imply that Monet was a Realist.
ration of the interior of a country, the existence and
Therefore, choice J is incorrect.
extent of lakes, valleys, and mountains, can often be
27. The correct answer is C. The first three paragraphs deal traced to the former prevalence of earthquakes and
with the gradual development of Impressionism, while volcanoes,” which supports choice C. The author states
the fourth paragraph deals with how it has been only that such cataclysms may have occurred in areas
unfairly judged. The author’s own descriptions of that are currently undisturbed, not that such undis-
Impressionist art indicates that it is an exciting form. So turbed areas are the direct causes of Earth’s features, so
there is sufficient support for choice C. Choice A only choice A is not the best answer. The author mentions

  896
www.petersons.com
Answer Keys and Explanations

land elevation in the paragraph, but does not indicate moral and political science, so the geologist should
that it is the cause of Earth’s features, so choice B is avail himself of the aid of many naturalists . . ., “ which
incorrect. Choice D may be true, but it is not something supports the conclusion in choice H. There is nothing
the author discusses in the passage, so it cannot be the about the difficulty of excavation in the passage, so
correct answer. choice F is incorrect. The passage does not imply that
geologists withhold information from each other
32. The correct answer is J. In the fourth paragraph, the
either, so choice G similarly lacks support. Choice J
author writes that “. . . a geologist should be well versed
refers to historians, not geologists.
in chemistry [statement III], natural philosophy,
mineralogy, zoology, comparative anatomy (statement 37. The correct answer is A. At the end of the third
II) [and] botany (statement I) … .” Therefore, the correct paragraph, the author writes that “. . . many distin-
answer should include all three statements, which is guishing features of the surface may often be
what choice J does. Choice F only accounts for one of ascribed to the operation, at a remote era, of slow and
the three true statements. Choices G and H each only tranquil causes—to the gradual deposition of
account for two of them. sediment in a lake or in the ocean, or to the prolific
increase of Testacea and corals,” which supports
33. The correct answer is C. In the sixth paragraph, the
choice A. This contradicts choice B. Increases in
author writes, “. . . at the close of the eighteenth century,
Testacea and coral are just examples of events that
geology appears to have been regarded as little other
may alter Earth’s surface, and there is no implication
than a subordinate department of mineralogy,” which
that one affects the other, so choice C is incorrect. The
supports choice C. Therefore, choices A and B cannot be
author merely uses an increase in coral as an example
true. Choice D seems to assume that the eighteenth
of a tranquil cause of Earth’s features and does not
century and 1800s are the same period.
imply that such a thing would make the Earth a more
34. The correct answer is F. The final sentence of the tranquil place (choice D).
passage specifies that “geology was in nowise con-
38. The correct answer is H. In the fifth paragraph, the
cerned ‘with questions as to the origin of things,’” so
author explains that “. . . the Egyptian mummies throw
choice F is the best answer. The author draws many
light on the art of embalming, the rites of sepulture,
comparisons between geology and history throughout
or the average stature of the human race in ancient
the passage, so choice G is not the best answer. At the
Egypt . . .,” which supports the conclusion in choice H.
beginning of the fourth paragraph, the author writes,
The author uses the study of coins, not mummies, as
“Geology is intimately related to almost all the physical
an example of ways to learn about rulers (choice F).
sciences,” which eliminates choice H. At the beginning
The author uses the study of ancient encampments,
of the second paragraph, the author writes that by
not mummies, as an example of way to discover the
studying geology, “. . . we acquire a more perfect
districts once occupied by armies (choice G). Choice J
knowledge of its present condition . . .,” which elimi-
is not implied in the passage; generally, mummifi-
nates choice J.
cation is a way to preserve the dead, not cure the
35. The correct answer is D. Each answer choice can be living.
used as a synonym for trace, but choice D makes the
39. The correct answer is D. The author states that
most sense in this particular context, because the
geologists, not historians, should have an under-
author is discussing finding and following a series of
standing of natural philosophy, so choice D is the best
events. Choices A and B refer to creating a drawing, not
answer. The author states that historians need
tracking events. The author uses trace as a verb, but
knowledge of ethics and politics (choice A) in
choice C suggests a noun.
Paragraph 4. In that same paragraph, the author
36. The correct answer is H. In the fourth paragraph, the explains that historians often rely on antiquaries,
author writes, “ . . . as such extensive acquisitions are which implies that historians must have working
scarcely within the reach of any individual, it is nec- relationships with other academics. He says they need
essary that men who have devoted their lives to insight into human affairs (choice C) in Paragraph 4.
different departments should unite their efforts; and as
the historian receives assistance from the antiquary, and
from those who have cultivated different branches of

  897
Peterson's ACT® Prep Guide 2018
Answer Keys and Explanations

40. The correct answer is G. In the second paragraph, the


author refers to such antecedent changes—violent past
events that affect the nations of today. Therefore, choice
G is the best example. Choice F can be eliminated
because it refers to geology rather than history. Choice
H is incorrect because current wars are not past events.
Choice J is incorrect because an unfortunate presi-
dential election is not as inherently violent as a
revolution.

  898
www.petersons.com
Answer Keys and Explanations

Science
1.   C 11.   D 21.   C 31.   B
2.   H 12.   F 22.   H 32.   F
3.   D 13.   C 23.   B 33.   D
4.   J 14.   G 24.   G 34.   J
5.   D 15.   A 25.   A 35.   D
6.   F 16.   G 26.   H 36.   J
7.   B 17.   A 27.   D 37.   D
8.   G 18.   J 28.   F 38.   H
9.   A 19.   C 29.   C 39.   B
10.   J 20.   H 30.   J 40.   J

1. The correct answer is C. From the table, at 4.5 V the Choices A, B, and C are incorrect because no
current for Resistor B is 420 mA at either temperature. additional information is necessary to conclude
Choice A would be correct if the question asked about that Resistor C is a thermistor.
Resistor C. Choice B would be correct if the question
6. The correct answer is F. Resistor A seems to be an
asked about Resistor C at 23°C. Choice D would be the
ohmic device because there is a linear relationship
current for Resistor A.
between voltage and current. If Resistor A released
2. The correct answer is H. Resistor A does not appear to heat into the water, it would not be considered an
be affected by external temperature. While 27°C is not ohmic device. Choice G is incorrect because
within the tested temperatures, it is a small change, and according to the table, Resistor B is not affected by
it’s reasonable to assume that Resistor A would show a temperature change. Choice H is incorrect because
similar current compared to the other tested tempera- temperature change would not affect this
tures. The current of Resistor A is linear with response to outcome. Choice J is incorrect because according
voltage, with 100 mA/volt. Therefore, 350 mA is the to the table, temperature does affect the current of
expected current at 3.5 V. Choice F is the expected Resistor C.
current of Resistor C at 23°C and 3.5 V. Choice G is the
7. The correct answer is B. The difference in
expected current of Resistor C at 25°C and 3.5 V. Choice
sediment bulk density between years 0 and 10 at
J is the approximate expected current of Resistor B at
Site 2 is 0.31 g/cm3. Choice A (between years 0 and
3.5 V.
4 at Site 1) is a difference of only 0.09 g/cm3. Choice
3. The correct answer is D. Both Resistor A and C behave C (between the two unplanted sites) is a difference
linearly, while Resistor B does not. The amount to which of only 0.08 g/cm3. Choice D (between the two
the current of Resistor C depends on voltage is different planted sites) is a difference of only 0.17 g/cm3.
at different temperatures, but in a linear fashion in both
8. The correct answer is G. The % nitrogen at the
cases. Therefore, choice A is insufficient, and choices B
10-year mark is more than 4 times greater than
and C are incorrect.
what was measured in the seagrass-free control.
4. The correct answer is J. At a given applied voltage, a The greatest change in Table 1 between years 0
thermistor produces different currents at different and 10 is the seagrass density; however that is not
external temperatures. Choices F and H do not address one of the choices. Choice F (% carbon) is less than
temperature at all. Choice G is incorrect because the twice as high at the 10-year measurement. Choice
definition of a thermistor does not depend on absolute H (% organic matter) is less than 1.5 times as high
currents produced, nor on comparison to other at the 10 year measurement. Choice J (bulk
resistors. density) decreases by approximately 20% at the
10-year measurement.
5. The correct answer is D. For a thermistor, a change in
current is observed when external temperature is 9. The correct answer is A. The ecologists cannot go
changed. At a given voltage with Resistor C, the back in time to test the original sediment compo-
observed current at 25°C is lower than that at 23°C. sition before seagrass planting, nor can they go
back to test the sediment of each replanted
  899
Peterson's ACT® Prep Guide 2018
Answer Keys and Explanations

meadow over time. The original sediment composition 13. The correct answer is C. The element with an atomic
under each of the two meadows may have been radius of 88 pm is krypton, which has a boiling point of
different, so it is important to collect a separate “year –153°C. Choice A is the boiling point of radon. Choice B
zero” for each of the meadows. Choice B is incorrect is the boiling point of xenon. Choice D is the boiling
because nothing in the passage indicates how sediment point of neon.
composition changes with water depth. Choice C is
14. The correct answer is G. Neon has an atomic weight of
incorrect because the ecologists have not experimen-
20.2 amu. Four times that weight is 80.8 amu, which is
tally controlled seagrass density in their experiments.
close to the atomic weight of krypton: 83.8 amu. Choice
Choice D is incorrect because the ecologists have not
F (helium) has an atomic weight of 4 amu, which is
conducted multiple measurements in unplanted
approximately one fifth that of neon. Choice H (xenon)
seabeds over time.
has an atomic weight of 131.3 amu, more than six times
10. The correct answer is J. Larger amounts of trapped that of neon. Choice J (radon) has an atomic weight of
sediment were found in areas with greater seagrass approximately 222 amu, which is more than ten times
surface area. The ecologists did not experimentally that of neon.
change one physical characteristic at a time, so Table 2
15. The correct answer is A. As the atomic number
looks at correlations, not causations. For the data to
increases from 2 to 86 down the column, the boiling
support each hypothesis, there should be a clear trend
point increases from –269 to–62°C. All boiling points in
between increasing sediment weight and an increase or
the table are below 0°C, but they approach zero as the
decrease of the seagrass characteristic. Choice F would
atomic number increases.
be correct if seagrass height increased consistently
down the table; however, there is no clear trend. Choice 16. The correct answer is G. The only pair of elements in
G would be correct if seagrass density decreased down the table that does not support the hypothesis is neon
the column; however, there is no clear trend. Choice H and argon (option I). All of the other pairs, including
would be correct if seagrass mass decreased down the both options II and III, support the hypothesis.
column; however there is no clear trend.
17. The correct answer is A. As the elements progress
11. The correct answer is D. The data in Table 1 shows little down the column, atomic number, atomic radius, and
or no change in % carbon and % nitrogen between 0 boiling point all increase. With an atomic number of
and 4 years, followed by significant change by 10 years. 118, oganesson lies below radon and therefore is
Choice A is incorrect because the passage has no record expected to have both a greater atomic radius and a
of the original seagrass density, nor how the replanted greater boiling point than those of radon.
areas compare. Choice B is similarly incorrect because
18. The correct answer is J. There is no systematic
there is no record of % organic matter in the original
relationship between thickness and depth of rock
meadow before seagrass removal. Choice C is incorrect
layers—the thickest layer (4) is found at less than 800 m,
because Table 1 shows little or no change in % carbon
and one of the thinnest layers (6) is below that. Choices
and % nitrogen between years 0 and 4.
F and G are incorrect. Choice H is incorrect because
12. The correct answer is F. The data in this passage layer 6 is 40 m thick, at depths of 960–1,000 m.
indicates that older seagrass meadows are more
19. The correct answer is C. Limestone is formed in marine
productive at storing carbon in the sediment than
environments, so the layers containing limestone give
younger meadows, at least up to (and perhaps beyond)
the age at which the region of the canyon was covered
10 years. Choice G could be correct if there was a
by sea. The time periods 130 million years and 220–260
demonstrated connection between added nutrients
million years contain limestone, but the latter time
and seagrass uptake and storage of atmospheric
period is not one of the choices. Choice A is incorrect
CO2—however, none of that is demonstrated in this
because there is no layer corresponding to 60 million
passage. Choice H is incorrect, since the data in Table 1
years ago. Choices B and D are incorrect because they
indicates that meadows 4 years or younger are less
do not contain limestone.
effective at storing carbon than those at least 10 years
old. Choice J is incorrect—while planting native plants 20. The correct answer is H. Choice F is incorrect because
along the shore may help overall ecosystem health, temperature is not listed at all in Figure 1. Choice G (age
there is no indication in the passage of how the of rock) is related to depth (the vertical axis), but that is
presence or absence of shore plants changes carbon not the measurement shown on the figure. Choice J
storage by the seagrass meadows.

  900
www.petersons.com
Answer Keys and Explanations

(thickness of rock layers) can be inferred from the figure, not directly support the position of a common ancestor.
but it is not the vertical axis. Examining theropod remains (choice D) does not relate
to Paleontologist A’s theory—he or she talks
21. The correct answer is C. The uranium ore (carnotite) is
about thecodonts.
found around petrified wood, which is found in layer 8.
That is at a depth of 1,110–1,270 m, which includes 26. The correct answer is H. Paleontologist B assumes that
1,200 m. Choice A is incorrect because it refers to the the body similarities between dinosaurs and early birds
thickness of layer 8, not the depth. Choice B is incorrect must be evidence that birds came forth from the
because layer 4 (500 m) does not contain petrified dinosaur lineage. This view does subscribe to the idea of
wood. Choice D is incorrect because there is no transitional forms. Mass extinction is not relevant to this
information for rock composition at 2,000 m. view. Choice F is incorrect because Paleontologist B
directly discusses transitional forms—shared features
22. The correct answer is H. The data does not show what
from immediate ancestors. Choice G is incorrect
range of ages are found in each layer, only the approx-
because extinction is not a main feature of
imate age difference between the different layers. The
Paleontologist A’s argument. Choice J is incorrect—
thickest layer (4) occurs between a separation of less
neither paleontologist thinks the lineages are distant.
than 20 million years, while one of the thinnest ones (3)
occurs between a separation of 40 million years. In 27. The correct answer is D. Paleontologist B believes that
reality, the thickness of a layer depends on a number of birds arose from a lineage of dinosaurs, which makes
factors, including the rate of sedimentation, the length choice C incorrect. Choice A is incorrect—Paleontologist
of time the region experienced a particular envi- B does not believe in convergent evolution. Choice B is
ronment, and the amount of weathering the rock layer the view of Paleontologist A.
experienced before being covered by a new layer.
28. The correct answer is F. This contradicts the theory of
23. The correct answer is B. Layers found at the top of the Paleontologist B, who suggests that birds arose from
canyon are youngest, so rocks at the surface were dinosaurs (making choices G and H incorrect).
deposited there less than the age of the highest Paleontologist A suggests that the two arose from an
layer—100 million years. The extreme temperatures extremely distant ancestor, and the theory of con-
necessary to form welded tuff are indicative of a vergent evolution is not inconsistent with birds
volcano. Choice A could be true, but a tropical climate is appearing before dinosaurs. Paleontologist A’s view is
not hot enough by itself to produce welded tuff. Choice present, so choice J is incorrect.
C is true based on the data in Table 1, but is not related
29. The correct answer is C. Paleontologist A postulates
to the existence of welded tuff. Choice D would be
the existence of a very distant common ancestor for
correct if the welded tuff was found at the lowest layer
birds and dinosaurs. The development of birds much
of the canyon, not the very surface.
later than that of dinosaurs might seem to refute this
24. The correct answer is G. Paleontologist A believes that argument. However, the rate of evolutionary change is
morphological (body) similarities can just as reasonably not constant across different lineages. Dinosaurs may
be assumed to represent the effects of convergent have developed relatively rapidly from thecodonts, for
evolution on distant lineages inhabiting similar example, whereas birds did not evolve until much later.
environments. Choice F is incorrect—this paleontologist Choice D is the opposite of this correct answer. Choices
does not mention a failed experiment of evolution. A and B are unrelated to the timing of the appearance
Choice H is incorrect because even though the paleon- of birds and dinosaurs.
tologist does not think birds evolved directly from
30. The correct answer is J. The dependent variable is the
dinosaurs, a connection is acknowledged. Choice J is
condition the experimenter is measuring throughout
incorrect—there is no discussion of what caused the
the experiment—in this case, that is the number of
thecodonts to survive extinction.
cultures with surviving paramecia. Choice F (the
25. The correct answer is A. Paleontologist A believes that number of species of bacteria) and choice G (the
dinosaurs and birds share a common ancestor. A fossil number of species of Paramecium) were both inde-
find from before the age of the dinosaurs with common pendent variables in this experiment. Choice H (the
features would support this view. An intermediate form length of time) could have been another independent
between dinosaurs and birds (choice B) or an avian variable if the researchers had examined the cultures at
prototype before dinosaur dominance (choice C) would

  901
Peterson's ACT® Prep Guide 2018
Answer Keys and Explanations

multiple points in time; since they were examined only 37. The correct answer is D. Mercury has an orbital
once, the length of time is a constant. inclination of 7.000 degrees and an orbital eccentricity
of 0.2056. Therefore, Mercury would be reclassified.
31. The correct answer is B. Examining the results in Tables
Choices A, B, and C are incorrect because none of these
2 and 3 shows that increasing the number of species of
planets have an orbital inclination greater than 5 or an
their food sources, the bacteria, did produce a relative
orbital eccentricity greater than 0.2.
increase in the survival rate of paramecia. Choice A
states the opposite, so it is incorrect. Choice C is 38. The correct answer is H. The orbital eccentricity of
incorrect because there is a definite increase. Choice D Venus is 0.0068; its orbit is the most circular in the solar
is incorrect—the only variable being observed is system. Mercury has an orbital eccentricity of 0.2056; its
paramecia, which increased. egg-shaped orbit exhibits the greatest eccentricity in
the solar system and thus deviates the most from a
32. The correct answer is F. Looking at Tables 2 and 3, it
circular orbit. Choice F is incorrect—there is a planet
appears that increasing the number of species of
with an orbital eccentricity closer to 0 than Earth.
Paramecium from one to two was most detrimental to
Choice G is incorrect—there is a planet with an orbital
their overall survival. The number decreased, making
eccentricity closer to 0 than Neptune. Choice J is
choice J incorrect. Choice G is incorrect because it
incorrect—there is a planet with an orbital eccentricity
cannot be confirmed—the only variable being
further from 0 than Neptune.
observed is paramecia. Choice H is incorrect—according
to Table 3, the number of species did have an effect on 39. The correct answer is B. The gaseous planets would be
paramecia survival. expected to be less dense than the ones made of solid
material. Choices A and D are incorrect—temperature
33. The correct answer is D. Experiment 3 added a
and rotation period should not be affected whether a
predator of paramecia. Choices A, B, and C are incorrect;
planet is a solid or a gas. Choice C is incorrect because
the number of bacteria species remained at 3, and we
these planets do not have greater masses than
have no information that the time or the culture
the others.
method was changed.
40. The correct answer is J. Mercury has the lowest mass
34. The correct answer is J. The results of this experiment
(3.3 x 1023 kg) and the highest orbital eccentricity
suggest that increasing diversity in terms of trophic
(0.2056°). The trend described in choice F is the opposite
levels was ultimately harmful to the survival of the
of what is observed; while not a perfect trend, more
Paramecium species under study. Choices F and G are
massive planets typically have shorter rotation periods.
incorrect; it was not beneficial to all species because of
Choice G is incorrect, because the two planets with the
the decreasing survival rates for the paramecia, and it
highest mass (Jupiter and Saturn) are not the two
definitely had an effect on paramecia. Choice H is
planets with the lowest densities (Saturn and Uranus).
incorrect, as we do not know the effect the experiment
Choice H would appear to be correct if the exponent in
had on Amoebae.
the mass term is ignored, since Earth and Mars have the
35. The correct answer is D. At 8.7 x 1025 kg, Uranus has the closest rotational period, less than an hour different. The
fourth greatest mass of the planets in the solar system. two planets with the closest relative mass (Uranus and
Jupiter, Saturn, and Neptune all have greater masses; Neptune) have rotational periods that differ by more
therefore, choices A, B, and C are incorrect. It is important than an hour.
to note the size of the exponent; otherwise, any of the
choices could appear correct.

36. The correct answer is J. Neptune has the lowest


temperature; based on that measure, it would seem to
be the farthest from the sun. Mercury (choice F), Uranus
(choice G), and Jupiter (choice H) all have higher
black-body temperatures and so are closer to the sun.

  902
www.petersons.com
Answer Keys and Explanations

Sample Essay: Score 1


Ideas and Analysis: Score = 1
Development and Support: Score = 1
Organization: Score = 1
Language Use and Conventions: Score = 1

I’m not sure I don’t use fossils for anything not something I care about even. I only plug in the stuff I used my phone and
my laptop and I think my TV they’re all plugged into the wall and I don’t bother anybody with them. Keep the sound
down low especially when its late and its late at night my parents say, so I do that and I know how to take care of my
stuff! If anybody wants to use anything they want to make theyre stuff work and as long as theyre not hurting anybody
its ok with me. If thyre hurting everybody they should stop and be nice people and use just the wall to plug theyre stuff
in, it is safe and I use it all the time and noone gets hurt so everybody should do that.

Scoring Explanation
Ideas and Analysis: Score = 1

This essay reflects a poor grasp of the essay task and a weak understanding of the role of fossil fuels and available energy sources.
As a result, ideas are scattered and confusing, the writer’s argument fails to address the questions provided, and the argument
veers off in a largely unrelated direction regarding his or her use of electricity and favorite electronic devices.

Development and Support: Score = 1

This argument suffers from a significant lack of development and support for a discernible perspective regarding the use of
alternative energy versus fossil fuels. This lack of idea development hinders comprehension and results in a disjointed essay
that does not address the essay task at hand.

Organization: Score = 1

The writer provides a disjointed set of loosely connected thoughts with little or no thought given to effective essay construction
or organization. There is no assertion of a clear perspective regarding the essay task, and the ideas provided don’t follow a
natural and coherent flow or support a unifying essay theme. The end result is a confusing piece of writing that falls off topic.

Language Use and Conventions: Score = 1

This essay demonstrates poor use of fundamental grammar and a deficient understanding of core English language rules. As a
result, this piece of writing, which is full of errors in usage and mechanics, fails to competently address the given essay task.

  903
Peterson's ACT® Prep Guide 2018
Answer Keys and Explanations

Sample Essay: Score 6

Ideas and Analysis: Score = 6


Development and Support: Score = 6
Organization: Score = 6
Language Use and Conventions: Score = 6

Let’s be honest. It’s nice to be able to drive to the beach on a warm summer day. It’s a wonderful luxury to have air conditioning
in our homes, heat during the winter, and enough electricity to power all of the electronic gadgets that make our lives more
comfortable. But is it worth any cost? Is it worth the destruction of our environment, the safety and well-being of future gener-
ations who have not yet had the opportunity to enjoy the natural wonders of our world, the very future of our planet? Each of
us can pretend that our use of energy doesn’t matter much, that it doesn’t have a huge impact on the Earth, but this is the sort
of apathetic mindset that keeps us from meaningful change.

The hard truth is that fossil fuels power our planet and are weaved into our social fabric; extricating ourselves from such a complex
intertwining won’t be easy. But that doesn’t mean it’s an issue we should avoid, or push off onto future generations. However,
calls for an immediate, rash, or unrealistic abatement of fossil fuel usage are unrealistic and prevent us from intelligently addressing
the challenge at hand.

What might a realistic, considered approach look like? There needs to be vastly increased incentives for individuals and businesses
of all sizes that are both utilizing renewable energy sources and working on innovative green energy solutions for society,
including tax breaks, monetary compensation, and government support. Towns, cities, and states need to galvanize and energize
their citizens, adopt greater alternative energy programs, and execute expanded information campaigns. Local and state gov-
ernments need to offer realistic strategies for reduced fossil fuel consumption and increased use of alternative energy that are
not cost prohibitive. Local, state, and federal budgets need to shift towards greater investment in this effort or we will surely
stall before we make any further progress.

Our country needs to keep energy consumption change at the forefront of our list of challenges to overcome. How can each of
us help make this happen? The power of the vote is a great place. We need to demand that our elected officials at all levels tackle
these issues—if they want to stay in office. Our collective concern can continue to be made known to our elected officials through
social media campaigns, petitions, and voting patterns. Nations around the world work together all the time and tackle tough
issues. Worldwide collaboration to effect positive climate change and reduce carbon emissions have already begin, but it needs
to move beyond its careful early steps and into larger and more audacious strategies. It’s too important an issue to not face it
bravely and boldly.

Here’s the bottom line: we need to figure out a realistic approach for decreasing fossil fuel reliance at every level—individuals,
towns, cities, states, countries, and nations working in unison can reduce non-renewable fuel consumption, advocate for and
invest in alternative fuel use, and—perhaps most importantly—acknowledge that we’re all responsible for the welfare of our
planet. Until we decide to make the hard decisions to effect positive change we’re placing our world in peril. Will it be easy?
Certainly not. But, the cost of continued complacency is much larger and more onerous—for each of us, for our children, and
for future generations.

  904
www.petersons.com
Answer Keys and Explanations

Scoring Explanation
Ideas and Analysis: Score = 6

The writer of this essay provides a well-reasoned, insightful argument in support of a realistic, unilateral approach to reducing fossil
fuel dependency and increasing investment, exploration, and use of alternative sources of fuel. The ideas are carefully considered
and represent a confident perspective on the issue. Progressive ideas for taking bolder steps are provided at every level—from
individuals to countries and political entities in-between, resulting in a cohesive, thoughtful piece of writing that clearly tackles
this issue with the seriousness it deserves.

Development and Support: Score = 6

This argument is a capably structured response to a challenging essay task, one that touches on hot-button social and political
issues and requires the writer to think on both micro and macro scales, which this response does quite well. From the provocative
opening question about fossil fuels (is it worth any cost?), through the detailed exploration of possible strategies for reducing our
non-renewable energy reliance and concluding plea for future generations, this essay demonstrates an impressive ability to develop
and support a clear perspective.

Organization: Score = 6

This argument is a well-organized, cogent piece of writing that serves the writer’s thoughts on the issue of alternative energy quite
well. It contains a clear perspective and sharp focus, from the mature and realistic introduction to the thought-provoking conclusion.
It does an especially admirable job of outlining what “ a realistic, considered approach” aligned with the writer’s thoughts on tapering
off from fossil fuels might look like.

Language Use and Conventions: Score = 6

This essay demonstrates an impressive command of standard language use and conventions, especially pertaining to argumentative
writing. The grammar and mechanics are strong throughout, and a wide array of engaging sentences and transitions are displayed.
The varied, thoughtful word choices and careful construction all lead to a thorough and effective essay that more than meets the
task at hand.

  905
Peterson's ACT® Prep Guide 2018

You might also like